You are on page 1of 104

1 Venous thromboembolism prophylaxis is not required in a:

A 76-year-old male requiring hip replacement surgery.

B 48-year-old male undergoing inguinal hernia repair.

C 35-year-old female underoging appendicectomy who is on oral contraception for 5 years.

D 30-year-old female undergoing abdomino-perineal resection for rectal cancer.

2 One of the following infection can lead to cholangiocarcinoma.


A Schistosomiasis

B Trypnosomiasis

C Strongyloidosis

D Clonorchis sinensis

3 Mixed tumours of the salivary gland are:


A associated with salivary calculi.

B usually malignant.

C the cause of facial paralysis.

D commonly seen in parotid gland.

4 Most of umbilical hernias in children


A resolve spontaneously.

B Usually get obstructed

C are associated with inguinal hernia.

D Always require elective surgery.

5 Concerning hypertrophic scar, it:


A outgrows it border.

B is an aberration in wound healing.

C tends to get worse even after a year.

D is treated by local steroid application.

6 One of the following feature is not that of a major pulmonary embolus.


A pleuritic pain.

B tachypnoea.

C peripheral cyanosis.

D evidence of right ventricular strain in ECG.

7 Signs of peritonits include all of the followings except:


A Percussion tenderness.

B Rebound tenderness.

C Guarding.

D Increased bowel sounds.

8 An elevated serum amylase is not see in:


A Perforated peptic ulcer.

B Acute hepatitis.

C Idiopathic pancreatitis.

D Mesenteric ischaemia.

9 In small bowel mesenteric ischaemia:


A Presence of intestinal gas is a common finding.

B Coelic axis is usually occluded.

C Mortality is less than 30% in acute event.

D A history of atrial fibrillation is common.

10 An incisional hernia is least likely to develop under the following situation.


A In patient with underlying malignancy.

B In severly septic patient.

C In transverse incisions.

D In patient with chronic obstructive airway disease.


11 In soft tissue injuries:
A Degloving injuries are best treated by simple suturing.

B Human bite wounds are heavily contaminated.

C Lacerations in hand due to broken glass or a knife are rarely associated with nerve or
tendon injuries.
D Haematoma should generally be left alone.

12 Basal cell carcinomas:


A spread by local direct invasions.

B frequently occurs on limbs.

C are very responsive to chemotherapy.

D generally grows rapidly.

13 Regarding gangrene
A Gangrenous tissue appears black owing to accumulation of melanin.

B Gas gangrene is caused by Clostridium perfringens.

C Diabetic gangrene is solely due to hyperglycaemia.

D Warming the affected part could be of therapeutic value.

14 The commonest post-operative complication following laparoscopic cholecystectomy is


A bowel perforation.

B portal vein injury.

C common bile duct injury.

D shoulder tip pain.

15 Regarding necrotising fasciitis:


A It is a specific bacterial infection.

B It involves mainly the skin, subcutaneous tissue and the deep fascia.

C Usually responds to antibiotics alone.


D Surgery should be avoided as far as possible.

16 A sebaceous cyst:
A Is occasionally seen in sole of the palm.

B Is an exudation cyst.

C Doesn’t have a capsule.

D Can lead to Cock’s peculiar tumor.

17 In undescended testis:
A The operation is recommended at the age of 5 years.

B Laparoscopy is the gold standard procedure for intra-abdominal testis.

C Orchidopexy reduces the chance of malignancy.

D It can be coaxed down into the scrotum.

18 In HIV (human immuno-deficiency virus) infection:


A The transimission risk is low during the stage of seroconversion.

B Most (HAART) anti-viral drugs act by stimulating to increase CD4 counts.

C The gut-associated lymphoid tissue is not affected.

D Universal precautions should be applied.

19 Antibiotic prophylaxis in endoscopy is not usually used in patient with:


A Previous history of endocarditis.

B Previous history of heart valves replacement.

C Previous history of cholecystectomy.

D Chronic liver disease undergoing variceal rubber banding.

20 The followings are problem associated with surgery in a jaundiced patient EXCEPT:
A Myocardial infarction.

B Hepatorenal syndrome.

C Clotting disorders.
D Infection.

1 A 76-year-old gentleman has passed only a small quantity of urine 24 hours after an inguinal
hernia surgery. He has a tender suprapubic mass on abdominal examination. The most
appropriate management will be:
A intravenous frusemide.

B add 1.5L of normal saline in addition to daily fluid requirement.

C add 1.5L of 5% dextrose in addition to daily fluid requirement.

D urinary catheterisation.

2 A 25-year-old male who is 15 hours’ post-laparotomy for a perforated chronic duodenal ulcer is
found to have a respiratory rate of 9 breaths/min. Physical examination shows neurological
poor response and reduced air entry in both lung fields. The likely diagnosis is:
A aspiration pneumonia.

B adult respiratory distress syndrome.

C narcotic overdose.

D pulmonary atelectasis.

3 On the fifth post-operative day after an emergency laparotomy, a patient was noted to have
serosanguineous discharge from the abdominal incision. The most appropriate treatment for the
problem is:
A Emergency re-exploration.

B Placing an abdominal binder.

C Intravenous antibiotics.

D Dressing of the wound.

4 A debilitated patient has undergone major upper gastrointestinal surgery. The most appropriate
postoperative nutritional support for him is:
A nasogastric feeding

B percutaneous endoscopic gastrostomy (PEG) feeding

C peripheral cannula parenteral feeding


D feeding jejunostomy

5 A 45-year-old man was admitted to general surgical ward with severe exacerbation of
ulcerative colitis. All necessary assessment was done and put up in next week operation list for
total colectomy. The BESTchoice of pre-operative nutritional support for him would be:
A nasojejunal feeding.

B nasogastric feeding.

C percutaneous endoscopic gastrostomy feeding.

D total parenteral nutrition.

1 In melanoma, the single most important prognostic index is:


A The size of the tumour.

B The distant metastasis.

C The satellite nodules.

D The presence of lymph node metastasis.

2 In determining the outcome of a patient with malignant melanoma the most important factor is:
A a history of pre-existing pigmented lesion.

B the site of the lesion.

C the width of excision of the lesion.

D the depth of invasion.

3 Which one of the following statement is true regarding tetanus?


A The incubation period is constant.

B Active immunization is only partially effective.

C The initial injury is often trivial.

D Outcome doesn’t depend on time of diagnosis.

1 A 38-year old man presenting with a left parotid tumour was explained about the complications
of surgery. All of the following are complications EXCEPT:
A great auricular nerve neuroma.
B gustatory sweating.

C cutaneous fistulae.

D contralateral gland hyperplasia

1) 30 year old lady comes with pain upper abdomen, fever, chills and rigors and with
icterus - what should be the first line of investigation ? :
A ) liver function test
B ) total haemogram
C ) ultrasound of hepato-biliary region
D ) CT scan of abdomen
2) 50 year old alcoholic, suddenly after meal, develops acute pain in abdomen with
fever, tachycardia, hypotension – likely diagnosis should be ? :
A ) acute gastro-enteritis
B ) acute pancreatitis
C ) acute cholecystitis
D ) acute mesenteric vascular ischaemia

3) 25 year old male reports with complaints of feeling heaviness of this right testis.
On examination, the testis is found to be hard in consistency with mild hydrocoele of
right scrotum. No inguinal lymphadenopathy and no history of trauma.
The preferred management should be :
A ) fine needle aspiration cytology (FNAC) of right testis
C ) open biopsy of right testis
D ) scrotal incision and removal of right testis
E ) inguinal incision and removal of right testis
1) Mammogram
A) detects breast carcinoma in young ladies less than 35 years old,
B) differentiates between solid lumps and cysts,
C) shows clustered microcalcifications in carcinoma breast,
E) is a painless procedure and can be repeated often.
2) Radiotherapy in carcinoma of breast
A) is routinely given after simple mastectomy for Paget’s disease,
B) can be given to contralateral breast as prophylaxis,
C) must be given after breast conserving surgery,
D) is safely given in pregnant ladies.

3) Grawitz’s Tumour of kidney,


A) is commonly seen in young children,
B) spreads early to regional lymph nodes,
C) is radiosensitive,
D) transabdominal nephrectomy is the main line of treatment.

4) Basal Cell Carcinoma :


A) is potentially curable with surgery,
B) distant metastases are found early in the disease,
C) spreads early in lymphatics,
D) dark-skinned ladies have a higher risk than fair-skinned men.

1. A ranula :
A is a malignant tumour of the oral cavity.

B metastasizes to the submandibular lymph nodes.

C may arise from the sublingual gland.

D is treated locally with radiotherapy.

2. Carcinoma of tongue :
A is usually an adenocarcinoma.

B commonly arises from the dorsum of tongue.

C presents initially with loss of taste before the tumour is visible.

D is related to chronic tobacco use.

3. In cholecystectomy, dissection of the Calot’s triangle is to identify and divide the :


A cystic duct.

B common bile duct.


C common hepatic duct.

D cystic artery.

4. The bitter taste of bile is due to :


A bicarbonate ions.

B bile salts.

C bile pigments.

D cholesterol.

5. Extrahepatic (or primary) biliary atresia :


A is hereditary.

B presents as prolonged neonatal jaundice.

C resolves spontaneously as the child grows up.

D is treated by exchange blood transfusion.

6. Choledochal cyst :
A is dilatation of the cystic duct.

B is caused by gall-stones.

C is treated by cholecystectomy.

D has increased risk of developing cholangiocarcinoma.

7. Important aetiological factors in hepatocellular carcinoma include :


A hepatitis A infection.

B hepatitis B infection.

C liver adenoma.

D amoebic abscess of liver.

8. The most common tumour marker in hepatocellular carcinoma is :


A alpha fetoprotein.

B Carcino-Embryonic Antigen (CEA).

C Carcinoma Antigen 15-3 (CA 15-3).

D Carcinoma Antigen 19-9 (CA 19.9).


9. Unresectable hepatocellular carcinoma is best treated with :
A external beam radiotherapy.

B intravenous 5-fluorouracil.

C transarterial chemoembolisation.

D ligation of hepatic artery.

10. The best screening tool for carcinoma of breast is :


A mammogram X-rays.

B serum Carcinoma Antigen 15-3 level (CA 15.3).

C oestrogen receptor status.

D serum Breast Cancer Type 1 Susceptibility Protein (BRCA - 1) levels.

11. The commonest cause of portal hypertension is :


A splenic vein thrombosis.

B liver cirrhosis.

C Budd-Chiari Syndrome.

D chronic pancreatitis.

12. In anal fistula, Goodsall’s rule is used to :


A predict the risk of malignant change.

B predict the risk of recurrence after surgery.

C determine the position of the internal opening.

D differentiate between “high” and “low” fistulae.

13. Which of the following is true about repair of groin hernias ?


A The Lichtenstein repair involves a multilayered imbricated repair of floor of inguinal
canal.
B The Shouldice repair uses a prosthetic mesh repair in inguinal canal floor in a tension-free
manner.
C The conjoint tendon is sutured to Cooper’s ligament in the Bassini repair.

D The McVay repair is suitable for repair of femoral hernias.

14. Direct inguinal hernias :


A are more common in women than men.

B are usually congenital.


C are more likely to strangulate that femoral hernias.

D can often be bilateral.

15. Swellings in the posterior triangle of the neck include :


A thyroglossal cyst.

B tuberculous lymphadenitis.

C goiter.

D branchial cyst.

16. Anal fissure :


A is a cause of severe pain on defaecation.

B is commonly located at 12 o’clock position of anus.

C is associated with passage of malaenic stools.

D is treated with steroid cream.

17. The most appropriate approach for a patient who has an enlarged prostate with a 5 cm diameter
bladder stone is :
A transurethral.

B transvesical.

C transperineal.

D retropubic.

18. Digital per rectal examination of a benign prostatic enlargement will reveal :
A obliteration of the median groove.

B adherence of the rectal mucosa to the prostate.

C gross asymmetry of the lateral lobes.

D smooth convexity of the posterior surface of the gland.

19 Complications of duodenal (peptic) ulcer include :


A obstructive jaundice.

B perforation.

C malignant change.

D polyps formation.
20. Injury to the ilio-inguinal nerve from abdominal incisions may result in :
A inguinal hernia.

B femoral henia.

C lumbar hernia.

D paraumbilical hernia.

1. A 55 year old diabetic man had Mayo’s repair of his paraumbilical hernia 4 days ago,
developed fever with redness, swelling and tenderness in an area surrounding the wound.
What would be the most appropriate treatment ?
A Continue oral antibiotics and observe.

B Open drainage of infection and debride any non-vital tissues.

C Increase his oral hypoglycaemic drug dosage.

D Prescribe analgesics, anti-pyretics and extended rest.

2. A 50 year old lady presented with an acute onset of high fever, jaundice and right
hypochondrial pain and tenderness but no palpable abdominal mass.
What is the most likely diagnosis ?
A Carcinoma head of pancreas.

B Acute pancreatitis.

C Perforated duodenal ulcer.

D Acute cholangitis.

3. A 45 year old man with a history of peptic ulcer presented with acute severe upper abdominal
pain, tenderness and board-like rigidity of whole abdomen with decreased bowel sounds.
What is the first immediate investigations to be ordered ?
A Plain X-ray abdomen in supine position.

B Blood and urine tests.

C Erect chest X-ray.

D Barium meal X-rays.

4. A 25 year old lady presented with a painless freely-movable 2 cm lump in her right breast.
The rest of the breast, the contralateral breast and axillae were normal.
What is the most appropriate management ?
A Excision biopsy of the lump.
B Right mastectomy.

C A trial course of antibiotics.

D Follow up 6 monthly monitoring

5. A 55 year old farmer noticed a hyperpigmented 1 cm diameter mole on his face increased in
size recently with itching. What is the most appropriate management ?
A Topical antifungal ointment.

B Topical antibiotic cream.

C Tru-cut core needle biopsy.

D Wide surgical excision biopsy extending down to subcutaneous tissue.

6. A 37 year old lady presented with a progressively enlarging hard lump in her left breast with no
nipple or skin changes. Fine Needle Aspiration Cytology confirmed a breast malignancy.
What is the most likely type of breast cancer ?
A Infiltrating ductal carcinoma.

B Infiltrating lobular carcinoma.

C Paget’s disease of breast.

D Inflammatory carcinoma of breast.

1. Malignant tracheo-oesophageal fistula from oesophageal carcinoma is best treated with :


A 3 stage radical oesophagectomy.

B radiotherapy.

C chemotherapy.

D an expanding metal stent.

2. Pain from bone metastases of breast carcinoma is best treated with :


A radiotherapy to the bone.

B anti-oestrogen drugs.

C surgical excision of bone metastases.

D chemotherapy.

3. Bilateral hydronephrosis due to an inoperable bladder carcinoma is treated with :


A bilateral nephrectomy.

B urinary diversion into an ileal conduit.


C intravesical chemotherapy.

D brachytherapy.

1. A 60 year old lady was diagnosed with carcinoma of the gallbladder.


Which of these signs would indicate advanced disease ?
A Jaundice.

B Large hard right hypochondrial mass.

C Ascites.

D All of the above.

1. Technetium 99m-labelled sestamibi isotope scan is a specific investigation for :


A papillary carcinoma of thyroid gland.

B parathyroid adenoma.

C gastric adenoma.

D medullary carcinoma of thyroid.

2. Which organ has a dual blood supply ?


A Brain.

B Liver.

C Stomach.

D Kidney.

3. In peptic ulcer disease, which of the following generally will perforate ?


A anterior duodenal ulcer.

B posterior duodenal ulcer.

C gastric ulcer at lesser curvature.

D gastric ulcer at antrum.

4. The commonest cause of haematemesis is :


A oesophageal malignancy.

B oesophageal varices.

C peptic ulcer disease.

D Mallory-Weis tear.
5. Which of the following blood groups is a risk factor for carcinoma of stomach ?
A A.

B B

C AB.

D O.

6. Krukenberg tumour of ovary seen in carcinoma of breast is due to :


A blood-borne metastases.

B lymphatic embolisation.

C peritoneal seedlings.

D direct spread.

7. The expected length of oesophagus in an adult is :


A 15 cm.

B 25 cm.

C 30 cm.

D 35 cm.

8. All the following are removed in Patey’s modified radical mastectomy, EXCEPT :
A pectoralis major.

B entire axillary content.

C entire breast with nipple-areolar complex.

D clavi-pectoral fascia.

1 Chvostek’s sign is seen in :


A. hypoparathyroidism.
B. hyperparathyroidism.
C. hypothyroidism.
D. hyperthyroidism.

2 Regarding carcinoma of the tongue :


A. it commonly involves dorsum of tongue.
B. it is usually an adenocarcinoma.
C. chemotherapy is the primary mode of treatment.
D. tobacco and alcohol are important aetiological factors.

3 Goodsall’s rule is used in anal fistula to :


A. predict the risk of malignant change.
B. determine the position of internal opening.
C. differentiate between “high” and “low” fistulae.
D. predict the risk of recurrence after surgery.

4 Parathyroid glands :
A. are normally palpable in the neck.
B. are exocrine glands.
C. can occasionally be found in the superior mediastinum.
D. produces calcitonin.

5 Surgical resection for carcinoma of caecum involves removing :


A. distal 20 cm of jejunum.
B. ileocolic artery and lymph nodes.
C. left colic artery and lymph nodes.
D. part of the inferior mesenteric artery.

6 Sistrunk operation :
A. is surgical excision of a ranula.
B. is done through intraoral approach without a neck incision.
C. involves removal of part of hyoid bone.
D. involves removal of anterior half of the tongue.

7 Thyroidectomy done for multinodular goiter involves removal of :


A. the whole thyroid gland..
B. recurrent laryngeal nerves.
C. all parathyroid glands.
D. all of the above.

8 The most common clinical presentation of carcinoma of rectosigmoid colon is :


A. haematochezia.
B. intussusception.
C. intestinal obstruction.
D. colonic perforation.

9 Most common site of colorectal carcinoma is :


A. sigmoid colon.
B. rectum.
C. transverse colon.
D. caecum.

10 Regarding papillary carcinoma of thyroid gland :


A. serum calcitonin level can be used as a tumour marker.
B. haematogenous spread is early.
C. usually arises from chronic multinodular goiter.
D. can initially present as an enlarged metastatic cervical lymph node.

11 Paget’s disease of breast usually presents as :


A. painful mobile breast lump.
B. progressively spreading eczema of nipple-areolar complex.
C. purulent nipple discharge.
D. diffuse inflamed swelling of breast.

12 Most common parotid gland tumour is :


A. pleomorphic adenoma.
B. adenolymphoma.
C. muco-epidermoid carcinoma.
D. acinic cell carcinoma.

13 Which malignancy is most likely to occur in long-standing multinodular goiter ?


A. Papillary carcinoma.
B. Follicular carcinoma.
C. Medullary carcinoma.
D. Anaplastic carcinoma.

14 Which of the following is true about repair of groin hernias ?


A. The conjoint tendon is sutured to Cooper’s ligament in the Bassini repair.
B. The McVay repair is suitable for repair of femoral hernias.
C. The Shouldice repair uses a prosthetic mesh repair in inguinal canal floor in a
tension-free manner.
D. The Lichtenstein repair involves a multilayered imbricated repair of floor of
inguinal canal.

15 In Grave’s disease, subtotal thyroidectomy is successful when there is an increase in :


A. serum thyroxine level.
B. serum tri-iodothyronine level.
C. serum calcium level.
D. serum Thyroid Stimulating Hormone level.

1 Breast conservative treatment for early breast carcinoma includes :


A. simple mastectomy.
B. adjuvant chemotherapy.
C. radiotherapy to chest wall.
D. radiotherapy to ipsilateral axilla.
2 The most common hernia in a child is a :
A. direct inguinal hernia.
B. indirect inguinal hernia.
C. femoral hernia.
D. obturator hernia.

3 The most common tumour of the parotid gland is the :


A. pleomorphic adenoma.
B. adenolymphoma (Warthin’s tumour).
C. ex-pleomorphic carcinoma.
D. acinic cell carcinoma.

4 A ranula :
A. is a tumour of the mucosa.
B. is malignant.
C. presents as a swelling under the tongue.
D. is treated with radiotherapy.

5 Stored blood transfusions :


A. are given for thrombocytopaenia.
B. can transmit Hepatitis A virus.
C. can transmit Hepatitis B virus.
D. are given when haemoglobin level is 12 mg/ 100 mls or less.

6 In hepatocellular carcinoma (HCC), which factor is most commonly present ?


A. A preceding Hepatitis A infection.
B. A preceding Hepatitis C infection.
C. Cirrhosis of liver.
D. Gallstones.

7 Squamous cell carcinoma :


A. commonly metastasizes by blood stream.
B. is common on the lower limb.
C. is associated histologically with keratin pearls.
D. is an autosomal recessive condition.

8 Malignant melanoma :
A. is the commonest skin malignancy.
B. is more commonly found in dark-skinned people.
C. always arises from pre-existing naevus.
D. carries poorer prognosis than basal cell carcinoma.

9 The most favoured approach in prostatectomy nowadays is :


A. transurethral.
B. transvesical.
C. retropubic.
D. transperineal.

10 Tumour marker for medullary carcinoma of thyroid gland is :


A. serum alphafetoprotein.
B. serum carcinoembryonic antigen.
C. serum calcitonin.
D. serum beta-human chorionic gonadotropin.

11 A hydrocoele in a 6 month old child is best treated with :


A. repeated aspiration.
B. excision of hydrocoele sac.
C. inguinal herniotomy.
D. conservative treatment.

12 Rovsing’s sign is elicited in :


A. acute appendicitis.
B. acute cholecystitis.
C. acute pancreatitis.
D. acute hepatitis.

13 Which of the following is most likely to cause a vesico-colic fistula ?


A. ischaemic colitis.
B. ulcerative colitis.
C. sigmoid diverticulitis.
D. carcinoma rectum.

14 Amoeboma :
A. is caused by Entamoeba coli.
B. is treated with metronidazole.
C. is most likely to affect the left lobe of liver.
D. is potentially malignant.

15 A 55 year old lady presents with a painless mobile lump in the upper outer quadrant of
her left breast for the past 1 month.
What would be the next appropriate step ?
A. Reassure her that the lump will resolve spontaneously with time.
B. Send her home with a course of antibiotics.
C. Do mammogram then needle aspiration for cytology.
D. Admit her for simple mastectomy.

16 A 7 month old baby had been crying on and off the whole day, then passed sticky dark
red stools. What is a likely diagnosis ?
A. Acute gastroenteritis.
B. Acute anal fissure.
C. Acute haemorrhoids.
D. Acute intussusception.

17 A 10 year old boy presents in the Emergency Department with acute severe painful
swelling of left scrotum. What is the likely diagnosis ?
A. Mumps orchitis.
B. Acute epididymoorchitis.
C. Acute testicular torsion.
D. Fournier’s gangrene.

18 Emergency circumcision is indicated in :


A. paraphimosis.
B. phimosis.
C. balanitis.
D. hypospadias.

19 A 27 year old man presents with a progressively enlarging hard swollen left testis of 2
months duration. What would be the most appropriate treatment ?
A. Fine needle aspiration cytology (FNAC) of right testis.
B. Open biopsy of right testis
C. Scrotal incision and removal of right testis..
D. Inguinal incision and removal of right testis.

20 Which area of the breast is least likely to be affected by breast cancer ?


A. Upper outer quadrant.
B. Upper inner quadrant.
C. Lower outer quadrant.
D. Lower inner quadrant.
21 Which of the following is most prone to develop carcinoma of testis ?
A. Epididymal cyst.
B. Ectopic testis.
C. Haematocoele.
D. Varicocoele.

22 Perthe’s test in lower limb varicose veins is to test for :


A. incompetence of sapheno-femoral junction.
B. incompetence of popliteal-femoral junction
C. incompetence of perforators.
D. patency of the deep veins..

23 The following is true of carcinoma of oesophagus :


A. Iinitially presents as difficulty in swallowing liquids.
B. Lower third tumour is treated by radiotherapy.
C. Metastases to cervical lymph modes carries poor prognosis.
D. Adenocarcinoma is more common in the upper third of oesophagus.

1 Marjolin ulcer is

A a type of acute gastric ulcer associated with major burn injury.

B an ulcer at an anastomosis after prior gastric surgery.

C a gastric vascular malformation.

D associated with squamous cell carcinoma in a chronic wound.

2 Dermoid cyst is

A commonly found in the mid line of the body.

B lined by columnar epithelium.

C adhered to the overlying skin.

D transillumination postive.
A

3 Sebaceous gland is

A apocrine type.

B holocrine type.

C merocrine type.

D endocrine type.

4 A 38 year-old lady presents to the emergency department with pain on defaecation lasting
about one hour. She also has noted some bright red blood on the tissue paper. The most
likely diagnosis is:

A Bleeding internal haemorhoids

B Thrombosed external haemorrhoids

C Proctitis

D Anal fissure

5 Single fistula-in-ano is commonly seen following

A Crohn’s disease.

B Colloid carcinoma of rectum.

C Perianal abscess.

D Tuberculosis

6 Vaginal hydrocoele is a type of:

A retention cyst.

B exudation cyst.

C cystic tumour.

D degenerative cyst.
B

7 The following conditions are congenital abdominal wall defects except

A Exomphalos minor

B Exomphalos major

C Umbilical hernia

D Ectopia Vesicae

8 An ulcer with an undermined edge is seen in

A gravitational ulcer.

B syphilitic ulcer.

C tuberculous ulcer.

D trophic ulcer.

9 Major cause regarding the mortality in congenital diaphragmatic hernia is due to:

A Size of the hiatus of the hernia.

B lung hypoplasia.

C presence of liver in the thoracic cavity.

D polyhydramnios.

10 62-year-old man with a history of smoking presents with hematuria. He has owned and
operated a chain of dry cleaners for over 30 years. The most likely diagnosis is:

A Prostate cancer

B Renal cell carcinoma

C Bladder cancer
D Carcinoma of the ureter
C

1. Severe head injury is considered to be having Glasgow Coma Scale (GSC) score of :
A. more than 12.
B. less than 10.
C. more than 10.
D. less than 8.

2. Blood discharge from the breast nipple is generally seen in :


A. breast abscess.
B. fibroadenoma.
C. duct papilloma.
D. cyclical mastitis (fibrocystic disease of breast).

3. The following factors may adversely affect wound healing, EXCEPT :


A. exposure to ultra-violet radiation.
B. jaundice.
C. advanced neoplasm.
D. uncontrolled diabetes mellitus.

4. Immediate post-operative pyrexia may occur secondary to, EXCEPT :


A. wound infection.
B. urinary tract infection.
C. deep vein thrombosis.
D. subphrenic abscess.

5. Pretibial myxoedema is a feature of :


A. lymphoma of thyroid.
B. myxoedema.
C. Grave’s disease.
D. Reidel’s thyroiditis.
6. Which is LEAST sensitive in detecting obscure gastro-intestinal bleeding ?
A. Barium meal and follow through X-ray.
B. Capsule enteroscopy.
C. Technetium-labeled radionuclide scan.
D. Selective angiography.

7. Complications of blood transfusion include the following, EXCEPT :


A. urticaria.
B. hypovolaemia.
C. Hepatitis C.
D. jaundice.

8. Which of the following is brilliantly transilluminant ?


A. sebaceous cyst.
B. dermoid cyst.
C. lipoma.
D. cystic hygroma.

9. Which of the following is an absorbable suture ?


A. polyglactin (Vicryl).
B. silk.
C. nylon.
D. steel wire.

10. Kehr’s sign is :


A. tenderness at hypochondrium in acute cholecystitis.
B. pain at right iliac fossa on pressing the left iliac fossa in acute appendicitis.
C. pain referred to left shoulder tip in ruptured spleen.
D. ecchymoses around umbilicus in ruptured ectopic pregnancy.

11. The most common organism responsidble for Fournier’s gangrene is :


A. Staphylococcus aureus
B. Escherischia coli.
C. Clostridium welchii.
D. Streptococcus pneumoniae.

12. Which of the following is decreased in acute pancreatitis ?


A. serum amylase.
B. serum lipase.
C. blood glucose.
D. serum calcium.

13. Cullen’s sign in seen in :


A. acute pancreatitis.
B. acute appendicitis.
C. acute cholecystitis
D. acute ischaemic colitis.

14. Perthe’s test is done to test for :


A. incompetence of sapheno-femoral junction.
B. patency of the deep veins.
C. incompetence of popliteal-femoral junction.
D. incompetence of perforators.

15. Portosystemic shunt can be done when :


A. serum albumin is more than 3 gm/ dl.
B. ascites is present.
C. patient is comatose.
D. hepatic encephalopathy is present.

16. The most common site of metastases of colorectal carcinoma is the :


A. right lung.
B. liver.
C. kidney.
D. brain.
17. The following is true of carcinoma of oesophagus :
A. metastases to cervical lymph modes carries poor prognosis.
B. lower third tumour is treated by radiotherapy.
C. initially presents as difficulty in swallowing liquids.
D. adenocarcinoma is more common in the middle third of oesophagus.

18. Risk factors for atherosclerosis includes the following, EXCEPT :


A. smoking.
B. diabetes mellitus.
C. high serum High Density Lipoprotein (HDL)
D. hypertension.

19. Pre-hepatic cause of portal hypertension includes :


A. Budd-Chiari Syndrome.
B. portal vein thrombosis.
C. alcoholic cirrhosis D. haemochromatosis

20. Barrett’s oesophagus is best diagnosed by :


A. ultrasound scan of abdomen.
B. barium meal and follow-through X-rays.
C. endoscopy and biopsy.
D. plain X-ray abdomen.

21. Marjolin ulcer is :


A. a type of acute gastric ulcer associated with major burns injury.
B. an anastomositic ulcer after gastric-intestinal surgery.
C. a variant of syphilitic ulcer.
D. a squamous cell carcinoma in a non-healing ulcer.

22. Sebaceous gland is a type of :


A. apocrine gland.
B. holocrine gland.
C. merocrine gland.
D. endocrine gland.
23. An ulcer with undermined edge is characteristic of :
A. gravitational ulcer.
B. syphilitic ulcer.
C. tuberculous ulcer.
D. trophic ulcer.

24. Basal cell carcinoma :


A. is highly malignant.
B. metastasizes early to regional lymph nodes.
C. is seen in any part of the face.
D. potentially curable with surgery.

25 Wilm’s tumour :
A. is seen in children.
B. is benign in origin.
C. initially presents with haematuria.
D. is treated conservatively.

26. Mammogram :
A. diffentiates between solid lumps and cysts.
B. is not a screening procedure.
C. showing clustered microcalcifications needs biopsy for carcinoma.
D. is painless and has bad effects of irradiation to breasts.

27. Amoeboma is :
A. a chronic amoebic liver abscess.
B. a malignant tumour associated with chronic amoebiasis.
C. a chronic granuloma arising mainly in the colon from amoebiasis.
D. mesenteric lymphadenitis resulting from intestinal amoebiasis.

28. Regarding full thickness burns of skin :


A. It is caused by hot water scalding.
B. The whole dermis is destroyed.
C. It presents as thin-walled painful blisters.
D. It heals completely without surgical treatment within 2 weeks.

29. A major cause of mortality in congenital diaphragmatic hernia is :


A. intestinal obstruction.
B. lung hypoplasia.
C. presence of liver in thoracic cavity.
D. polyhydramnios.

30. The following are congenital abdominal wall defects, EXCEPT :


A. exomphalos minor.
B. exomphalos major.
C. umbilical hernia.
D. ectopia vesicae.

31. Which of the following is true of thyroglossal cyst :


A. It originates from the thyroid gland.
B. It is found in the lateral aspect of the neck.
C. It is a premalignant condition.
D. Its removal involves resection of part of hyoid bone.

32. A 32 year old truck driver presents with a painful fluctuant swelling in his gluteal cleft
for 2 days. What is the most likely diagnosis ?
A. Ischiorectal abscess.
B. Pilonidal abscess.
C. Crohn’s disease of anus.
D. tuberculous ulcer.

33. Femoral hernias :


A. are more common in men.
B. are less likely to strangulate than inguinal hernias.
C. are found medial to the femoral vein.
D. occur as a result of infection.
34. A 25 year old motorcyclist involved in an accident, sustained severe facial jnjuries with
deformity and bleeding . Which of the following measures is correct ?
A. Evaluation of cervical spine should precede the facial injuries management.
B. Direct oral or nasotracheal intubation should performed promptly to prevent airway
obstruction.
C. Standard facial X-ray series are preferable to CT scan to evaluate facial fractures.
D. Definitive management of fractures of facial bones should not be delayed.

1. Severe head injury is considered to be having Glasgow Coma Scale (GSC) score of :
A. more than 12.
B. less than 10.
C. more than 10.
D. less than 8.

2. Blood discharge from the breast nipple is generally seen in :


A. breast abscess.
B. fibroadenoma.
C. duct papilloma.
D. cyclical mastitis (fibrocystic disease of breast).

3. The following factors may adversely affect wound healing, EXCEPT :


A. exposure to ultra-violet radiation.
B. jaundice.
C. advanced neoplasm.
D. uncontrolled diabetes mellitus.

4. Immediate post-operative pyrexia may occur secondary to, EXCEPT :


A. wound infection.
B. urinary tract infection.
C. deep vein thrombosis.
D. subphrenic abscess.

5. Pretibial myxoedema is a feature of :


A. lymphoma of thyroid.
B. myxoedema.
C. Grave’s disease.
D. Reidel’s thyroiditis.

6. Which is LEAST sensitive in detecting obscure gastro-intestinal bleeding ?


A. Barium meal and follow through X-ray.
B. Capsule enteroscopy.
C. Technetium-labeled radionuclide scan.
D. Selective angiography.

7. Complications of blood transfusion include the following, EXCEPT :


A. urticaria.
B. hypovolaemia.
C. Hepatitis C.
D. jaundice.

8. Which of the following is transilluminant ?


A. sebaceous cyst.
B. dermoid cyst.
C. lipoma.
D. cystic hygroma.

9. Which of the following is an absorbable suture ?


A. polyglactin (Vicryl).
B. silk.
C. nylon.
D. steel wire.

10. Kehr’s sign is :


A. tenderness at hypochondrium in acute cholecystitis.
B. pain at right iliac fossa on pressing the left iliac fossa in acute appendicitis.
C. pain referred to left shoulder tip in ruptured spleen.
D. ecchymoses around umbilicus in ruptured ectopic pregnancy.

11. The most common site of metastases of colorectal carcinoma is the :


A. right lung.
B. liver.
C. kidney.
D. brain.

12. Which of the following is decreased in acute pancreatitis ?


A. serum amylase.
B. serum lipase.
C. blood glucose.
D. serum calcium.

13. Cullen’s sign is seen in :


A. acute appendicitis.
B. acute cholecystitis.
C. acute pancreatitis.
D. acute ischaemic colitis.

14. Perthe’s test is done to test for :


A. incompetence of sapheno-femoral junction.
B. patency of the deep veins.
C. incompetence of popliteal-femoral junction.
D. incompetence of perforators.

15. Femoral hernias :


A. are more common in men.
B. are less likely to strangulate than inguinal hernias.
C. are found medial to the femoral vein.
D. occur as a result of infection.

16. Which of the following is true of thyroglossal cyst :


A. It originates from the thyroid gland.
B. It is found in the lateral aspect of the neck.
C. It is a premalignant condition.
D. Its removal involves resection of part of hyoid bone.

17. The following is true of carcinoma of oesophagus :


A. metastases to cervical lymph modes carries poor prognosis.
B. lower third tumour is treated by radiotherapy.
C. initially presents as difficulty in swallowing liquids.
D. adenocarcinoma is more common in the middle third of oesophagus.

18. Risk factors for atherosclerosis includes the following, EXCEPT :


A. smoking.
B. diabetes mellitus.
C. high level of serum High Density Lipoprotein (HDL)
D. hypertension.

19. Pre-hepatic cause of portal hypertension includes :


A. Budd-Chiari Syndrome.
B. portal vein thrombosis.
C. alcoholic cirrhosis.
D. haemochromatosis.

20. Barrett’s oesophagus is best diagnosed by :


A. ultrasound scan of abdomen.
B. barium meal and follow-through X-rays.
C. endoscopy and biopsy.
D. plain X-ray abdomen.

21. Marjolin ulcer is :


A. a type of acute gastric ulcer associated with major burns injury.
B. an anastomositic ulcer after gastric-intestinal surgery.
C. a variant of syphilitic ulcer.
D. squamous cell carcinoma in a chronic non-healing ulcer.

22. The following are congenital abdominal wall defects, EXCEPT :


A. exomphalos minor.
B. exomphalos major.
C. umbilical hernia.
D. ectopia vesicae.

23. An ulcer with undermined edge is characteristic of :


A. gravitational ulcer.
B. syphilitic ulcer.
C. tuberculous ulcer.
D. trophic ulcer.

24. Basal cell carcinoma :


A. is highly malignant.
B. spreads early to regional lymph nodes.
C. is treated primarily with radiotherapy.
D. potentially curable with surgery.

25 Wilm’s tumour :
A. is seen in children.
B. is benign in origin.
C. presents initially with haematuria.
D. is treated conservatively.

26. Mammogram :
A. diffentiates between solid lumps and cysts.
B. is not a screening procedure.
C. showing clustered microcalcifications needs biopsy for carcinoma.
D. has bad effects of irradiation on the breasts.

27. Amoeboma is :
A. a chronic amoebic liver abscess.
B. a malignant tumour associated with chronic amoebiasis.
C. a chronic granuloma arising mainly in the colon from amoebiasis.
D. mesenteric lymphadenitis resulting from intestinal amoebiasis.

28. A major cause of mortality in congenital diaphragmatic hernia is :


A. intestinal obstruction.
B. lung hypoplasia.
C. presence of liver in thoracic cavity.
D. polyhydramnios.

29. A 32 year old truck driver presents with a painful fluctuant swelling in his gluteal cleft
for 2 days. What is the most likely diagnosis ?
A. Ischiorectal abscess.
B. Pilonidal abscess.
C. Crohn’s disease of anus.
D. Prolapsed haemorrhoids.

30. A 25 year old motorcyclist involved in an accident, sustained severe facial jnjuries with
deformity and bleeding . Which of the following measures is correct ?
A. evaluation of cervical spine should precede the facial injuries management.
B. direct oral or nasotracheal intubation should performed promptly to prevent airway
obstruction.
C. standard facial X-ray series are preferable to CT scan to evaluate facial fractures.
D. definitive management of fractures of facial bones should not be delayed.

1 The portal vein :


A. is the union of splenic vein with superior mesenteric vein.
B. supplies 25% of blood to the liver.
C. has semilunar valves.
D. can be ligated and divided in the repair of liver lacerations.

2 The least common site of colorectal carcinoma is :


A. rectum.
B. sigmoid colon.
C. transverse colon.
D. caecum.
3 Familial Adenomatous Polyposis :
A. is also known as juvenile polyp.
B. has a high risk of malignancy.
C. affects only the colon.
D. affects only males.

4 Bilateral gynaecomastia is seen in patients with :


A. Turner’s Syndrome.
B. diabetes mellitus.
C. tuberculosis.
D. cirrhosis of liver.

5 Phyllodes tumour of breast :


A. is benign.
B. commonly affects young girls.
C. is small, soft and cystic.
D. usually undergoes malignant transformation to carcinoma.

6 Malignancy in a solitary thyroid nodule can be confirmed by :


A. thyroid function tests.
B. fine needle aspiration cytology.
C. ultrasound scan of the nodule.
D. radioisotope scan.

7 Grave’s Disease :
A. is associated with multinodular goiter.
B. is treated by total thyroidectomy.
C. is commonly seen in young ladies.
D. is best investigated by radioisotope scan.

8 Regarding Thyroid Stimulating Hormone;


A. it is secreted by the posterior pituitary gland.
B. its secretion is suppressed by high circulating thyroid hormones.
C. stimulates the secretion of calcitonin from the thyroid gland.
D. is decreased in all cases of hypothyroidism.

9 The gallbladder :
A. concentrates bile.
B. conjugates bilirubin
C. converts bilirubin to biliverdin.
D. produces bile.

10 Regarding primary biliary atresia ;


A. all patients have jaundice at birth.
B. is directly linked to a genetic abnormality.
C. is a congenital malformation of bile ducts.
D. if untreated, patients die of liver failure.

11 The Sengstaken-Blakemore tube is used to stop bleeding in :


A. gastric ulcers.
B. duodenal ulcers.
C. oesophageal varices.
D. epistaxis.

12 Most appropriated treatment for pleomorphic adenoma of parotid gland is :


A. simple enucleation of tumour.
B. superficial parotidectomy.
C. radical parotidectomy with facial nerve reconstruction.
D. local radiotherapy.
1 Who is considered the “Father of Modern Medicine” ?
A. Sushruta.
B. Hippocrates.
C. Galen.
D. Lister.
2 Maturation of scar takes place in :
A. 0 to 4 weeks.
B. 4 to 12 weeks.
C. 12 to 40 weeks.
D. 40 to 60 weeks.

3 Rapid transfusion of stored blood may produce :


A. hypothermia.
B. hypercalcaemia.
C. dark stools.
D. tea-coloured urine.

4 Cushing’s Syndrome is typically caused by :


A. adrenocortical hyperplasia.
B. adenoma of posterior lobe of pituitary gland.
C. adenoma of anterior lobe of pituitary gland.
D. hyperaldosteronism.

5 Which of the following is associated with pituitary tumours ?


A. Insulinoma.
B. Bilateral hemianopia.
C. Episodes of watery diarrhoea.
D. Hypokalaemia.

6 Episodic hypertension and glycosuria is associated with :


A. Type 1 diabetes mellitus.
B. insulinoma.
C. Multiple Endocrine Neoplasia (MEN) Type1.
D. phaechromocytoma.

7 Treatment with oral iron sulphate for anaemia results in :


A. haemolytic jaundice.
B. pale coloured stools.
C. indigestion.
D. spherocytic red blood cells.

8 In acute arterial occlusion of a limb :


A. it is characterized by a warm limb.
B. treatment is immediate use of vasodilator drug.
C. irreversible muscle necrosis occurs after 6 hours.
D. urgent whole blood transfusion is given.

9 An ulcer by definition is :
A. a defect caused by infection.
B. a bleeding malignant lesion.
C. an abrasion of the skin.
D. a breach in the continuity of an epithelial surface.

10 Barrett’s oesophagus is :
A. a premalignant lesion of the oesophagus.
B. is a frank malignant lesion of the oesophagus
C. is due to a viral infection.
D. is due to duodenal ulceration.

11 Regarding the thyroid gland :


A. the superior laryngeal nerves runs alongside the lateral border of the gland.
B. it is continuous with the thymus.
C. the recurrent laryngeal nerve is close to the inferior thyroid artery.
D. it is normally palpable in the posterior triangle of the neck.

12 Thyroid gland hypofunction may present clinically with :


A. bradycardia.
B. weight gain despite increased appetite.
C. fine hand tremors.
D. pre-tibial myxoedema.

13 Regarding medullary carcinoma of thyroid :


A. it usually occurs in families.
B. it produces high level of serum calcitonin.
C. it produces high level of tri-iodothyronine.
D. it arises from follicular cells.

14 Sebaceous cyst :
A. has a punctum.
B. is found on palms of hands.
C. is found in the oral mucosa.
D. will turn malignant.

15 Factors that promote wound healing include :


A. Vitamin C deficiency.
B. Good blood supply.
C. Irradiation.
D. Steroid therapy.
1 Mammogram :
A. is not a screening procedure.
B. showing clustered microcalcifications needs biopsy for carcinoma.
C. differentiates between solid lumps and cysts.
D. is painless and has does not involve ionizing irradiation.

2 Femoral hernias :
A. are more common in men.
B. are less likely to strangulate than inguinal hernias.
C. are found medial to the femoral vein.
D. occur as a result of infection.

3 Choledochal cyst :
A. is dilatation of the cystic duct.
B. is caused by gall-stones.
C. is treated by cholecystectomy.
D. has increased risk of developing cholangiocarcinoma.

4 Cullen’s sign in seen in :


A. acute pancreatitis.
B. acute appendicitis.
C. acute cholecystitis
D. acute ischaemic colitis.

5 Kehr’s sign is :
A. ecchymoses around umbilicus in ruptured ectopic pregnancy.
B. pain at right iliac fossa on pressing the left iliac fossa in acute appendicitis.
C. pain referred to left shoulder tip in ruptured spleen.
D. tenderness at hypochondrium in acute cholecystitis

6 Which of the following is brilliantly transilluminant ?


A. Cystic hygroma.
B. Sebaceous cyst.
C. Lipoma.
D. Dermoid cyst.
7 Haemorrhoids
A. is always associated with carcinoma of rectum.
B. may present with dark tarry stools.
C. can cause hypochromic microcytic anaemia.
D. is always treated by open haemorrhoidectomy.

8 Which of the following is decreased in acute pancreatitis ?


A. Serum amylase.
B. Serum lipase.
C. Blood glucose.
D. Serum calcium.

9 Pretibial myxoedema is a feature of :


A. medullary carcinoma thyroid.
B. Grave’s disease.
C. lymphoma of thyroid.
D. Reidel’s thyroiditis.

10 The following factors may adversely affect wound healing, EXCEPT :


A. jaundice.
B. advanced neoplasm.
C. exposure to ultra-violet radiation.
D. uncontrolled diabetes mellitus.

11 Pre-hepatic cause of portal hypertension includes :


A. alcoholic cirrhosis.
B. portal vein thrombosis.
C. Budd-Chiari Syndrome.
D. haemochromatosis.
12 The most common site of metastases of colorectal carcinoma is the :
A. liver.
B. right lung.
C. kidney.
D. brain.

13 Portosystemic shunt can be done when :


A. patient is comatose.
B. ascites is present.
C. serum albumin is more than 3 gm/ dl.
D. hepatic encephalopathy is present.

14 Upper gastrointestinal bleeding is best diagnosed by :


A. plain X-ray abdomen.
B. barium meal and follow-through.
C. ultrasound scan.
D. fibreoptic endoscopy.

15 Complications of blood transfusions include :


A. Hepatitis A transmission.
B. jaundice.
C. hypovolaemia.
D. hypercalcaemia.

MCQ :

1: B 2 : C 3: D 4: A 5: C

6 : A 7 : C 8 : D 9 : B 10 : C
11 : B 12 : A 13 : C 14 : D 15 : B

1.The following colonic polyps are pre-malignant EXCEPT


a. Juvenile polyps
b. Hamartomatous polyps associated with Peutz-Jeghers syndrome
c. Villous adenomas
d. Tubular adenomas

2. All of the following are true regarding tetanus EXCEPT


a. The organism produces a powerful endotoxin
b. The toxin prevents the release of inhibitory neurotransmitter
c. Clostridium tetani is sensitive to penicillin
d. Risus sardonicus is the typical facial spasm

3. One of the following statements is TRUE regarding small bowel obstruction


a. In the United Kingdom is most often due to an obstructed hernia
b. Causes colicky abdominal pain and vomiting
c. Abdominal distension is seen in all patients
d. All cases can be managed conservatively for the first 24 hours

4. All of the following statements are true regarding bladder calculi EXCEPT
a. The incidence has fallen markedly in this country since the late 19th century
b. They may be totally asymptomatic
c. They are more prevalent in patients with chronic urinary sepsis
d. They increase the risk of transitional cell carcinoma

5. All of the following statements are true in obstructive jaundice EXCEPT


a. Urinary conjugated bilirubin is increased
b. Urinary urobilinogen is reduced
c. Serum conjugated bilirubin is reduced
d. Faecal stercobilinogen is reduced

6. The treatment of choice in a 6 month old child with congenital hydrocele is


a. Excision of hydrocele
b. Aspiration
c. Herniotomy
d. Conservative management

7. Rovsing’s sign is elicited in


a.Acute appendicitis
b. Acute cholecystitis
c.Acute pancreatitis
d. Diverticulitis

8. Dumping syndrome is usually seen after


a. Oesophageal surgery
b. Gastric surgery
c. Small bowel surgery
d. Colonic surgery

9.Which of the following conditions is most likely to cause a vesico-colic fistula?


a. Cancer of the rectum.
b. Ulcerative colitis.
c. Sigmoid diverticulitis.
d. Ischaemic colitis.

10. The symptoms of chronic duodenal ulcer are


a. most often experienced in the early hours of the morning.
b. Unremitting—continuous.
c. Associated with right shoulder pain.
d. Associated with a poor appetite.

1. A 28 year old man is brought to the Emergency Department with a history of skidding and
falling off his motorcycle 30 minutes ago. He complains of severe breathlessness and chest
pain. There is no history of loss of consciousness. On examination, the respiratory rate is 38/
minute, pulse rate is 110 per minute and BP is 90/60mmHg. The JVP is raised. The right
chest seems to be moving less than the left and there is no air entry on the right chest on
auscultation.. There is no obvious rib fracture clinically. The SpO2 reading is 88%. The
immediate step to be taken to save life is
a. Ambu bag ventilation
b. Intubation and ventilation
c. Needle thoracostomy
d. Cricothyroidotomy
e. Colloid infusion

2. A 30-year old motorcyclist was admitted to the ward after a closed reduction and above knee
plaster cast for fracture shaft of tibia. 12 hours after admission you were called to see him
for severe unceasing pain in his injured leg. You would
a. elevate his leg in a Braun’s frame
b. prescribe him pethidine injection to relieve the pain
c. split the plaster cast completely
d. send him for a check x-ray to see if there is any re-displacement of the fracture
e. reassure him
3. A 38 year old lady had undergone hysterectomy for menorrhagia. In the post-operative period
her paO2 level was found to be decreasing in spite of delivering oxygen through high flow
nasal mask. Her ventilation/perfusion ratio was found to be abnormal. The probable cause
would be
a. lower abdominal surgery
b. starvation
c. increased cardiac output
d. upright position in bed
e. pulmonary embolism
4. A man had a cerebral concussion after an accident. A few days later he complained of vertigo
whenever he turned his head. It was very short lived but at times associated with nausea and
vomiting. His hearing was normal. What is the diagnosis ?
a. Acute vestibular failure
b. Vestibular neuronitis
c. Benign paroxysmal positional vertigo
d. Labyrinthitis
e. Endolymphatic hydrops

5. A 25-year old farmer presents with a painful red eye. He had sustained injury to the eye with
plant material while working in his field a week ago. Eye examination shows conjunctival
discharge and ciliary congestion. There is a 3-mm sized hazy lesion in the cornea which
stains with fluorescein. Pupil is 3 mm and reacting. Vision is 6/18. The most probable
diagnosis is
a. acute iridocyclitis
b. acute congestive glaucoma
c. muco purulent conjunctivitis
d. Corneal ulcer
e. Interstitial keratitis

A 4-year-old boy is referred by his general practitioner with an enlarged left kidney. He is otherwise
asymptomatic and completely well. He is afebrile. Urinalysis reveals microscopic haematuria. Urine
cultures are negative. The most likely diagnosis is
(a) nephroblastoma
(b) renal cell carcinoma
(c) transitional cell carcinoma
(d) adenoma Kidney

One of the following is strong indication for prostatectomy in benign prostatic hyperplasia.
(a) very large prostate on PR examination
(b) acute retention of urine with raised urea in blood
(c) frequency
(d) difficult to initiate micturition

Which one of the following mechanisms is associated with posterior urethral rupture?
(a) forceful catheterization
(b) fall astride on the projecting object
(c) pubic bone fracture
(d) iatrogenic injury during anterior resection of rectum

Under water sealed intercostal tube:


(a) is the recommended treatment for all pneumothorax
(b) is best performed through 2nd intercostal space mid-axillary line for pneumothorax
(c) should aim towards to the apex for the drainage of pneumothorax
(d) should be connected with high pressure suction machine in tension pneumothorax

Amoeboma is
(a) a chronic granuloma arising mainly in the colon
(b) a chronic amoebic liver abscess
(c) mesenteric lymphadenitis resulting from intestinal amoebiasis
(d) a malignant tumour associated with chronic amoebiasis
Which area of the breast is least likely to be affected by breast cancer?
(a) Upper inner quadrant.
(b) Lower outer quadrant.
(c) Lower inner quadrant.
(d) Upper outer quadrant.

A 13-year-old student presents with bilateral tender and swollen breasts. He has become increasingly
self-conscious of late and has been absent in physical education classes. On examination there are
tender soft masses in the lower
quadrants of both breasts. What is the likely diagnosis?
(a) Acute mastitis.
(b) Fibroadenoma.
(c) Lymphoma.
(d) Gynaecomastia

Out of the followings, which carcinoma of breast usually carry poorer prognosis?
(a) Colloid carcinoma
(b) Tubular carcinoma
(c) Inflammatory carcinoma
(d) Medullary carcinoma

Ionizing radiation may cause the following malignancies.


(a) Carcinoma of thyroid
(b) Carcinoma of breast
(c) Lymphoma
(d) Mesothelioma

The most important transplant antigen is


(a) blood group A antigen
(b) Human Leukocyte Antigen (HLA)
(c) Rhesus antigen
(d) blood group B antigen

A 23-year-old pregnant woman presents with constipation and bright red blood
coating her stools. On examination, two bluish tender spongy masses are found
protruding from the anus. These do not reduce spontaneously and require digital
reduction. The most likely diagnosis is:
(a) anal carcinoma
(b) rectal prolapse
(c) haemorrhoids
(d) peri-anal warts

One of the following is prone to develop carcinoma of testis.


(a) Epididymal cyst
(b) Haematocoele
(c) Undescended testis
(d) Ectopic testis

Normal value of prostate specific antigen (PSA) is:


(a) 0-4 ng/ml
(b) 0-10 ng/ml
(c) 0-20 ng/ml (d) 0 -30ng/ml
Common benign tumour found in kidney is:
(a) Adenoma
(b) Oncocytoma
(c) Angiolipoma
(d) Haemangioma

Nephro-ureterectomy is the operative procedure of choice in the:


(a) Adenoma carcinoma of the kidney,
(b) Angiolipoma of the kidney,
(c) Transitional carcinoma of pelvis and kidney,
(d) Adenoma of the kidney.

All cause the bloody discharge from the nipple of the breast except:
(a) Intraductal papilloma.
(b) Paget’s disease of the breast.
(c) Intraductal carcinoma of the breast.
(d) Duct ectasia.

Drug not used in the treatment of prostatic carcinoma is:


(a) luteinizing hormone-releasing hormone (LH-RH) agonists.
(b) Stilboesterol
(c) Cyproterone acetate
(d) Testosterone

Malignant tumour arising from the kidney and bladder could be:
(a) Squamous cell carcinoma.
(b) Transitional cell carcinoma.
(c) Adenocarcinoma.
(d) All of the above.

Palliative surgery done for inoperable peri-ampullary carcinoma of the pancreas is:
(a) Stenting.
(b) Cholecysto-jejunostomy.
(c) Choledocho-jejunostomy.
(d) All of the above.

Regarding the Glasgow Coma Scale, the followings are true except:
(a) It is used to assess the severity of head injury.
(b) A high score correlates with high mortality.
(c) It measures the motor response.
(d) It measures verbal response.

The most common initial manifestation of increasing intra-cranial pressure in a victim of head
trauma is:
(a) Change in level of consciousness.
(b) Ipsilateral pupillary dilatation.
(c) Contralateral pupillary dilatation.
(d) Hemiparesis.

Treatment of gas gangrene includes all of the following except:


(a) Administration of penicillin.
(b) Administration of hyperbaric oxygen
(c) Administration of antitoxin.
(d) Wound debridement.
The commonest cause of rectal bleeding in the children is:
(a) Meckel’s diverticulum
(b) Intussusception
(c) Fissure in ano
(d) Intestinal polyps

A man was brought to the emergency department of a local hospital with a painful swelling in his left
scrotum following an injury while playing baseball. The treatment of choice is:
(a) Rest and icebag application.
(b) Rest and hot water bag application.
(c) Scrotal support and analgesic.
(d) Surgical exploration.

In colorectal cancers:
(a) At least 50% are in the rectum and sigmoid colon.
(b) Low anterior resection is easier in males than in females.
(c) Most 1cm polyps contain a focus of cancer.
(d) Alteration in bowel habit is common in right sided cancers.

The normal ankle brachial pressure index (ABPI) is:


(a) 0.5
(b) 1
(c) 1.5
(d) 2

Hypersplenism includes all of the following except:


(a) Splenomegaly
(b) Pancytopaenia
(c) Bone marrow hyperplasia
(d) Splenic vein thrombosis

A 43-year-old policeman presents with a painless ulcer over the left medial malleoli with
surrounding pigmentation. The ulcer is likely to be:
(a) Ischaemic ulcer
(b) Venous stasis ulcer
(c) Marjolin’s ulcer
(d) Diabetic ulcer

All of the following factors will delay wound healing except:


(a) Malnutrition
(b) Irradiation
(c) Steroid therapy
(d) Good blood supply

The followings are features of sebaceous cyst except:


(a) Presence of Punctum.
(b) Presence of Sebaceous horn.
(c) Tethering to the skin.
(d) Erosion of the underlying bone
A 25-year-old woman presents to the emergency room complaining of redness
and pain in the right leg. She reports that her right leg has been swollen for the
last 5 years. On examination, her left leg has been normal. She has temperature of 39C and right leg
is not tender but swollen from the inguinal ligament down and she has obvious cellulitis. Her
underlying problem is
(a) Popliteal entrapment syndrome
(b) Acute arterial insufficiency
(c) Primary lymphoedema
(d) Deep vein thrombosis

In full thickness burns of skin:


(a) the whole dermis is destroyed.
(b) it is caused by hot water scalding.
(c) thin-walled painful blisters are usually present.
(d) it heals completely without surgical treatment within 2 weeks.

Burn contractures occur in:


(a) partial thickness burn
(b) whole thickness burn
(c) scalds
(d) frost bite

Regarding medullary carcinoma of thyroid it:


(a) arises from follicular cells.
(b) produces high level of serum calcitonin.
(c) usually occurs in families.
(d) produces high level of tri-iodothyronine.

In hypothyroidism:
(a) pre-tibial myxoedema is present.
(b) it is associated with low serum Thyroid Stimulating Hormone levels.
(a) it occurs immediately after total thyroidectomy.
(b) it occurs immediately after radioiodine therapy

In diffuse toxic goiter :


(a) fine needle aspiration cytology(FNAC) is the investigation of choice.
(b) serum Thyroid Stimulating Hormone is raised.
(c) measurement of serum calcitonin is essential.
(d) oral anti-thyroid drugs can be started immediately upon diagnosis.

Papillary carcinoma of thyroid with lymph node involvement is best treated with :
(a) total thyroidectomy with cervical lymph node dissection.
(b) radio iodine ablation of remnant thyroid tissue.
(c) long-term TSH suppression with oral thyroxine.
(d) all of the above.

A 70 year old lady presents with a progressive enlarging painless hard thyroid nodule, hoarseness of
voice and weight loss. What is the likely diagnosis?
(a) multinodular goitre.
(b) anaplastic carcinoma of thyroid.
(c) follicular adenoma.
(d) de Quervain’s thyroiditis.
1 Gas gangrene :
A. is seen in traumatic wounds of head and neck.
B. is caused by gram negative bacilli.
C. can be prevented by active immunization.
D. may present with crepitus in the wound.

2 One feature which is NOT associated with acute appendicitis is


A. Zachary Cope’s sign.
B. Rovsing’s sign.
C. maximum tenderness at Mayo-Robson’s point.
D. peri-umbilical pain later shifted to right iliac fossa.

3 Congenital diaphragmatic hernias


A. present with respiratory distress in the neonatal period.
B. are more common on the right than the left.
C. are rarely associated with other congenital anomalies.
D. are due to hypoplastic lungs.

4 Regarding intussusception in children, it:


A. usually occurs in the jejunum.
B. usually presents with few clinical signs.
C. has a characteristic 'doughnut' appearance on ultrasound.
D. is usually due to a polyp.

5 The most common tumour in the appendix is:


A. argentaffinoma.
B. lymphoma.
C. adenocarcinoma.
D. gastro-intestinal stromal tumour (GIST).
6 In tuberculous lymphadenitis of the neck:
A. the glands are rubbery.
B. supraclavicular group is commonly affected.
C. surgical removal is the treatment of choice.
D. chronic sinus may form.

7 All of the following factors have increased risk of surgical site infection, EXCEPT :
A. positive pressure airflow in the operating theatre.
B. hypothermia during surgery.
C. skin shaving.
D. perioperative poor blood glucose control.

8 All the following have high risk of developing malignancy by tobacco, EXCEPT :
A. urinary bladder.
B. oesophagus.
C. lungs.
D. liver.

9 Stridor is present in all, EXCEPT :


A. retropharyngeal abscess.
B. tracheolaryngobronchitis.
C. acute epiglottitis.
D. acute asthmatic attack.

10 Prophylactic antibiotics should be given


A. at the time of bowel anastomosis..
B. immediately after skin incision.
C. immediately after induction.
D. one hour before surgery.
11 Haemorrhoids
A. may present with dark tarry stools.
B. can cause hypochromic microcytic anaemia.
C. is always associated with carcinoma of rectum.
D. is always treated by haemorrhoidectomy.

12 Hartmann’s solution does NOT contain:


A. sodium.
B. potassium.
C. bicarbonate.
D. chloride.

13 Carcinoma of the stomach is NOT associated with:


A. linitis plastica.
B. volvulus of the stomach.
C. Troisier’s sign.
D. blood group ‘A’.

14 An amoebic liver abscess:


A. is an end result of amoebic hepatitis.
B. is caused by Entamoeba coli.
C. yields yellowish pus on aspiration.
D. can be excluded if stool examination of parasite yields negative.

15 A 45-year-old female presents with a painful and red perianal swelling at 9 o’clock
position with fever for 3 days. The best option of treatment is:
A. systemic antibiotics and analgesics.
B. Sitz baths.
C. emergency surgery.
D. topical application of antibiotics and local analgesics.
In fibreoptic colonoscopy the following cannot be visualized :
A : lumen of appendix .
B : ileocaecal junction.
C : lumen of Meckel’s diverticulum.
D : diverticulum of colon.

In Endoscopic Retrograde Cholangio-Pancreaticography, the procedure uses :


A : a side-viewing flexible duodenoscope.
B : an end-viewing flexible gastroscope.
C : a flexible colonoscope.
D : a laparoscope.

In Oesophago-Gastro-Duodenoscopy, the procedure is usually done :


A : after giving laxatives to the patient one day prior to procedure.
B : after the patient has had a full meal.
C : under general anaesthesia.
D : when the patient has fasted overnight.

Endoscopic Retrograde Cholangio-Pancreaticography can be used therapeutically for :


A : retrieving stone from common bile duct.
B : retrieving stones from gall-bladder.
C : reducing venous pressure in portal hypertension.
D : aspiration of pseudocyst of pancreas.

Minor salivary glands :


A : are paired and few in number in the oral cavity.
B : are palpable on the dorsum of the tongue
C : give rise to mucocoele of lower lip.
D : get infected and ulcerate to form aphthous ulcers.
Pleomorphic adenoma of parotid gland :
A : starts as a painless swelling on the cheek.
B : commonly infiltrates the facial nerve causing facial palsy.
C : is caused by cigarette smoking.
D : surgery by simple enucleation of tumour is the preferred treatment.

Ranula :
A : is a malignant tumour of the tongue.
B : is palpable as a hard lump in the posterior triangle of neck.
C : require post-operative radiotherapy to control spread.
D : is none of the above.

Stone in the submandibular duct :


A : can be palpable along the duct just under the tongue.
B : is usually asymptomatic and need no surgical intervention.
B : causes Frey’s Syndrome.
C : can dissolve with using repeated saline gargle.

Carcinoma of the tongue usually arise from :


A : the posterior third.
B : the dorsum.
C : deep within the muscle layers.
D : lateral aspects.

Commonest site of occurrence of adenocarcinoma of colon is :


A : transverse colon.
B : sigmoid colon.
C : hepatic flexure.
D : splenic flexure.
Measures that can help to reduce risk of developing adenocarcinoma of colon, include :
A : vegetarian diet
B : a diet rich in animal fats.
C : tobacco smoking.
D : alcohol consumption.

Wilm’s tumour of kidney :


A : occurs in elderly men.
B : is benign.
C : is a malignancy seen in children.
D : is radiosensitive.

Grawitz’s tumour of kidney :


A : is a malignancy seen in children.
B : nephrectomy is usually done via loin approach.
C : can present initially as secondaries in lungs or bone.
D : usually presents with retention of urine.

Regarding urinary calculi :


A : all can be visualized by cystoscopic examination.
B : open surgery is the only option available to remove all stones.
C : are mostly radioopaque and can be seen on plain abdominal X-rays.
D : tend to grow very large before being symptomatic.

Clinical features of obstructive jaundice include :


A : bilous vomiting.
B : light coloured stools.
C : light coloured urine.
D : constipation.
The normal thyroid gland :
A : secretes through thyroglossal duct.
B : is palpable in anterior triangle of neck.
C : has two lobes connected by isthmus behind the trachea.
D : moves up on swallowing because of its connection to the tongue.

In full thickness burns of skin :


A) the whole dermis is destroyed.
B) is caused by hot water scalding.
C) usually presents as thin-walled superficial painful blisters.
D) heals completely with non-surgical treatment within 2 weeks.

Features of medullary carcinoma of thyroid include :


A) arise from follicular cells of thyroid gland.
B) serum calcitonin can be used as a tumour marker.
C) usually occur in families.
D) hypertension if present is due to high thyroxine levels.

Hypothyroidism is associated with :


A) pre-tibial myxoedema.
B) is associated with low serum Thyroid Stimuating Hormone levels.
C) occurs immediately after total thyroidectomy.
D) occurs immediately after radioiodine therapy.

In diffuse toxic goiter :


A) Fine Needle Aspiration Cytology is the investigation of choice.
B) elevated serum Thyroid Stimulating Hormone is expected.
C) measurement of serum Calcitonin is essential.
D) Oral anti-thyroid drugs can be started immediately upon diagnosis.
Papillary carcinoma of thyroid with lymph node involvement is best treated with :
A) total thyroidectomy with cervical lymph node dissection.
B) radioiodine ablation of remant thyroid tissue.
C) long-term TSH suppression with oral thyroxine.
D) all of the above.

A 70 year old lady presents with a progressive enlarging painless hard thyroid nodule,
hoarseness of voice and weight loss. What is the likely diagnosis ?
A) diffuse toxic goiter.
B) anaplastic carcinoma of thyroid.
C) follicular adenoma.
D) de Quervain’s thyroiditis.

1. A 75 year old man presents with persistent right upper quadrant pain.
He is a known case of advanced carcinoma caecum with secondaries in the liver.
He is in atrial fibrillation and on Warfarin treatment.
What would be your strategy for symptom relief ?
A) Liver transplantation.
B) Chemotherapy with cis-platinum.
C) Radiofrequency ablation.
D) Radiotherapy to the liver.

2. A 75 year old man presents with watery diarrhoea and evidence of multiple
metastases in the liver.
He is cachexic, hyokalaemic with hypochlorhydria.
The level of vasoactive intestinal polypeptide is very high and a tumour of
the pancreas is suspected. cachetic.
What is your management?
A) Codeine phosphate.
B) Pancreatectomy
C) Radiofrequency ablation of liver lesions.
D) Somatostatin treatment

1. In a trauma patient assessment of an unobstructed air way is


A. if a patient can talk with no hoarseness
B. if a patient has stridor
C. If therer isIntercostal retraction during inspiration
D. If a patient is gasping
2. The indication for intubation in a coma patient is if
A. Glasgow coma score of 8 or less
B. Trauma to the neck with hoarseness
C. No stridor
D. Glasgow coma score of 13 or more

Treatment of pneumothorax of the right chest is


A. Place chest tube in the 5th inter costal space in midaxillary line
B. Insert chest tube through the inferior boarder of the rib
C. Connect chest tube to under water seal
D. Connect chest tube to the suction

4. In hemo pneumo thorax thorocotomy indicated if there is


A. Bleeding from the chest tube is 50cc an hour
B. Bleeding for the chest tube is 200cc an hour
C. If the lung is collapsed due to pneumo throax
D. Bleeding from the chest tube 100cc an hour

5 Signs of Tension pneumo thorax are the following except


A. Muffled heart sounds
B. Trachea deviated to the opposite side
C. Engorged neck veins
D. Blood pressure of 80/60 mmHg

6 In a class II shock patient signs of adequate resuscitation includes


A. tachycardia
B. Urine out put of 30 – 40cc per hour
C. Presence of lactic acidosis
D. Venous oxygen saturation of 90%
7 Cushing’s sign of lateralization in a closed head injury occurs due to
A. hypotension
B. Brain edema and ischemia
C. Epidural hematoma
D. Linear fracture of skulls

8 In a 18 year old male was in a car crash and found to have no respiration may be due to
A. Fracture of 6th cervical spine
B. Fracture of 2nd cervical spine
C. Fracture of 7th cervical spine
D. Depressed fracture of the skull

9. In a 46 year old male involved in a auto mobile accident, has on chest X-ray widened
mediastinum with indistinct aortic knob. The diagnosis is
A. Pneumo thorax
B. Ruputured aorta
C. Rib fractures
D. Right pleural effusion

10. A patient who is 25 year old fell from a height 20feet c/o pain abdomen. Indication for
Laprotomy is
A. DPLReveals100,000 RBC/ml or WBC7,500
B. Laceration of abdominal wall 2inches
C. No rebound tenderness of abdomen
D. Bowel sounds present

11. A 15year old male with gun shot wound to abdomen on laprotomy has a hole in sigmoid
colon. Best choice of treatment is
A. Suture the hole in colon with non absorbable suture
B. Segmental resection of colon with end to end anastomosis
C. Exteriorization of colon with repair after 6weeks
D. Primary repair and give antibiotics.
20. A patient in coma following a car crash has a distended abdomen. The evaluation of the
abdominal trauma is by
A. Ultra sound
B. CT scan
C. DPL ( diagnostic peritoneal lavage)
D. Rectal exam

13. A14 year old boy is hit over the side of the head with a base ball bat. He loses
consciousness for a few minutes and recovers promptly and continues to play. He is
found in the locker room unconscious. Right pupil is dilated and fixed.
A. Sub arachnoid hemorrhage
B. Acute epidural hematoma
C. Basal skull fracture
D. Subdural hematoma

14. If a patient sustains a penetrating wound in the middle of neck. The management is
A. Pressure dressing over the wound
B. Immediate surgical exploration
C. angiography
D. CT scan of the neck

15. A four year old child had a gun shot injury to the arm. He is hypotensive with
tachycardia. Three attempts to start peripheral IV have failed. What is the next step
A. Central line
B. Interaosseous cannulation
C. IV cut down
D. Subcutaneous injection of saline

16. A 22year old boy brought to the ER following a gun shot wound to the chest. He is pale
diaphoretic, cold and shivering with a BP 60/40. Pulse 150/mt. Distended neck veins and
there is no breath sounds on the right chest. The diagnosis is
A. Tension pneumothorax
B. Cardiac tamponade
C. Hemo pneumothorax D.flail chest
17. A 67 year old male fell of a horse a month ago, prior to which he was active. At present
he stares at the wall, barely talks and has altered sensorium. The diagnosis is
A. Stroke (CVA)
B. Chronic sub dural hematoma
C. Depressed skull fracture
D. Intra cerebral hemorrhage

18. A 55 year old man involved in a car crash. Examination reveals no breath sounds in the
left side of the chest. Percussion is unremarkable. Chest X- ray shows multiple fluid
levels in the left chest. The diagnosis is
A. Hemo pneumothroax
B. Gastric rupture
C. Diaphragmatic rupture
D. Liver laceration

19. A 27 year old intoxicated man crashes against a tree. He has tenderness in the left chest
wall. X-ray reveals fracture ribs 89 and 10. BP is 80/60. Pulse 128/mt. The diagnosis is
A. Small bowel rupture
B. Splenic rupture
C. Pancreatic injury
D. Rupture of the left kidney

20. A 25 year old man involved in high speed auto collision has pelvic fracture with fresh
blood at the urethral meatus. The initial evaluation of the patient includes
A. Ultra sound
B. CT scan
C. Retrograde urethrogram
D. Rectal exam
1. A 8. B 15. B

2. A 9. B 16. A

3. A 10. A 17. B

4. B 11. C 18. C

5. A 12. C 19. B

6. B 13. B 20.

7. B 14. B
1. Right gastric artery is normally a branch of
A. Celiac axis artery
B. Common hepatic artery
C. Gastroduodenal artery
D. Superior mesenteric artery

2. The most common cause of peritonitis is


A. duodenal ulcer perforation
B. perforation of carcinoma of colon
C. tuberculosis of peritoneum.
D. gall bladder perforation

3. The most common cause of painful haematuria is


A. carcinoma of urinary bladder
B. renal cell carcinoma
C. infection of urinary tract
D. oxalate stones in urinary tract

4. Prehepatic cause of portal hypertension is


A. Budd- Chiari syndrome
B. Wilson’s disease
C. Portal vein thrombosis
D. Alcoholic cirrhosis

5. Direction of movement of mesenteric cyst on clinical examination is


A. upwards and downwards
B. sideward
C. parallel to plane of posterior attachment of mesentry
D. in a plane at right angle to the line of attachment of mesentry

6 . Enlargement of spleen occurs mostly towards


A. left iliac fossa
B. right iliac fossa
C. left dome of diaphragm
D. right dome of diaphragm

7. Following are the causes of Dysphagia EXCEPT


A. carcinoma oesophagus
B. scleroderma
C. oesophageal varices
D. achalasia cardia
8. A male presents after motor vehicle accident with fracture pelvis and few drops of blood being
passed per urethra. He has inability to pass urine. The bladder is palpable. Most likely cause is
A. renal trauma
B. transected ureter
C. bladder rupture
D. urethral injury
9. Common bile duct stone is associated with all of the following EXCEPT
A. serum alkaline phosphatase is raised in common bile duct obstruction
B. cholangitis will cause Charcot’s triad in clinical picture
C. they are often associated with palpable gall bladder
D. pancreatitis is a complication of bile duct stone

10. Pseudocyst of pancreas most commonly occurs after


A. trauma
B. pancreatitis
C. Pancreatic surgery
D. pancreatic malignancy

1. A 60 year old man is admitted with abdominal pain, vomiting, distention and constipation for the
past one week. He gives a history of appendicectomy done 2 years ago. Provisionally he is diagnosed
to have “intestinal obstruction”. Which one of the following is the initial investigation of choice?
A. CT scan
B. MRI
C. Plain X-ray
D. Barium enema.

2. A 25 year old woman suffers impact injuries following a car accident. She is apparently doing well
till the 2nd day when she begins to vomit and complains of shortness of breath. Plain Xray taken in
the Emergency room shows loss of diaphragmatic contour. What is the most likely cause?
A. Ruptured Gallbladder.
B. Ruptured Liver
C. Ruptured Spleen
D. Ruptured Diaphragm.

3. A 60 year old male who is chronic smoker is diagnosed recently suffering from bronchogenic
carcinoma. He is found to be very irritable, tired and confused. He has lost his appetite. He also
states that he is breathless and feels thirsty and wakes up several times at night to pass urine. The
most underlying cause is
A. Lung collapse
B. Cerebral hypoxia
C. Para neoplastic syndrome
D. Acute glomerulonephitis.

1. The dioptric power of the cornea is: ( D )


A) 12d c) 33d
B) 25d d) 44d
2. Night blindness occurs in: ( C )
A) Optic neuritis
B) Anterior uveitis
C) Retinitis pigmentosa
D) Vitamin C deficiency

3. Sympathetic ptosis is caused by ( C )


A) paralysis of oribicularis oculi
B) paralysis of levator palpebrae superioris
C) paralysis of mullers muscle
D) paralysis of oblique muscle

4. Which of the following intra-ocular foreign-bodies causes Chalcosis : ( A )


A) Copper
B) Lead
C) Iron
D) Gold

A 70-year old lady, known hypertensive complains of sudden loss of vision in left eye since
morning; he was apparently normal till yesterday. The eye is quiet. Pupil shows RAPD. Fundus
exam shows normal optic disc; retinal arteries are markedly narrowed; retina is milky-white but
fovea shines as a bright red spot. Veins show 'cattle-truck' appearance. What is your diagnosis ?
A) Central Retinal vein occlusion
B) Papilloedema
C) Central Retinal artery occlusion
D) Hypertensive retinopathy Ans : C
A 4-year-old boy is referred by his general practitioner with an enlarged left kidney. He is otherwise
asymptomatic and completely well. He is afebrile. Urinalysis reveals microscopic haematuria. Urine
cultures are negative. The most likely diagnosis is
(e) nephroblastoma
(f) renal cell carcinoma
(g) transitional cell carcinoma
(h) adenoma Kidney
One of the following is strong indication for prostatectomy in benign prostatic hyperplasia.
(e) very large prostate on PR examination
(f) acute retention of urine with raised urea in blood
(g) frequency
(h) difficult to initiate micturition

Which one of the following mechanisms is associated with posterior urethral rupture?
(e) forceful catheterization
(f) fall astride on the projecting object
(g) pubic bone fracture
(h) iatrogenic injury during anterior resection of rectum

Under water sealed intercostal tube:


(e) is the recommended treatment for all pneumothorax
(f) is best performed through 2nd intercostal space mid-axillary line for pneumothorax
(g) should aim towards to the apex for the drainage of pneumothorax
(h) should be connected with high pressure suction machine in tension pneumothorax

Amoeboma is
(e) a chronic granuloma arising mainly in the colon
(f) a chronic amoebic liver abscess
(g) mesenteric lymphadenitis resulting from intestinal amoebiasis
(h) a malignant tumour associated with chronic amoebiasis

Which area of the breast is least likely to be affected by breast cancer?


(e) Upper inner quadrant.
(f) Lower outer quadrant.
(g) Lower inner quadrant.
(h) Upper outer quadrant.

A 13-year-old student presents with bilateral tender and swollen breasts. He has become increasingly
self-conscious of late and has been absent in physical education classes. On examination there are
tender soft masses in the lower
quadrants of both breasts. What is the likely diagnosis?
(e) Acute mastitis.
(f) Fibroadenoma.
(g) Lymphoma.
(h) Gynaecomastia

Out of the followings, which carcinoma of breast usually carry poorer prognosis?
(e) Colloid carcinoma
(f) Tubular carcinoma
(g) Inflammatory carcinoma
(h) Medullary carcinoma

Ionizing radiation may cause the following malignancies.


(e) Carcinoma of thyroid
(f) Carcinoma of breast
(g) Lymphoma
(h) Mesothelioma
The most important transplant antigen is
(e) blood group A antigen
(f) Human Leukocyte Antigen (HLA)
(g) Rhesus antigen
(h) blood group B antigen

A 23-year-old pregnant woman presents with constipation and bright red blood
coating her stools. On examination, two bluish tender spongy masses are found
protruding from the anus. These do not reduce spontaneously and require digital
reduction. The most likely diagnosis is:
(e) anal carcinoma
(f) rectal prolapse
(g) haemorrhoids
(h) peri-anal warts

One of the following is prone to develop carcinoma of testis.


(e) Epididymal cyst
(f) Haematocoele
(g) Undescended testis
(h) Ectopic testis

Normal value of prostate specific antigen (PSA) is:


(d) 0-4 ng/ml
(e) 0-10 ng/ml
(f) 0-20 ng/ml
(g) 0-30 ng/ml

Common benign tumour found in kidney is:


(e) Adenoma
(f) Oncocytoma
(g) Angiolipoma
(h) Haemangioma

Nephro-ureterectomy is the operative procedure of choice in the:


(e) Adenoma carcinoma of the kidney,
(f) Angiolipoma of the kidney,
(g) Transitional carcinoma of pelvis and kidney,
(h) Adenoma of the kidney.

All cause the bloody discharge from the nipple of the breast except:
(e) Intraductal papilloma.
(f) Paget’s disease of the breast.
(g) Intraductal carcinoma of the breast.
(h) Duct ectasia.

Drug not used in the treatment of prostatic carcinoma is:


(e) luteinizing hormone-releasing hormone (LH-RH) agonists.
(f) Stilboesterol
(g) Cyproterone acetate
(h) Testosterone
Malignant tumour arising from the kidney and bladder could be:
(e) Squamous cell carcinoma.
(f) Transitional cell carcinoma.
(g) Adenocarcinoma.
(h) All of the above.

Palliative surgery done for inoperable peri-ampullary carcinoma of the pancreas is:
(e) Stenting.
(f) Cholecysto-jejunostomy.
(g) Choledocho-jejunostomy.
(h) All of the above.

Regarding the Glasgow Coma Scale, the followings are true except:
(e) It is used to assess the severity of head injury.
(f) A high score correlates with high mortality.
(g) It measures the motor response.
(h) It measures verbal response.

The most common initial manifestation of increasing intra-cranial pressure in a victim of head
trauma is:
(e) Change in level of consciousness.
(f) Ipsilateral pupillary dilatation.
(g) Contralateral pupillary dilatation.
(h) Hemiparesis.

Treatment of gas gangrene includes all of the following except:


(e) Administration of penicillin.
(f) Administration of hyperbaric oxygen
(g) Administration of antitoxin.
(h) Wound debridement.
The commonest cause of rectal bleeding in the children is:
(e) Meckel’s diverticulum
(f) Intussusception
(g) Fissure in ano
(h) Intestinal polyps

A man was brought to the emergency department of a local hospital with a painful swelling in his left
scrotum following an injury while playing baseball. The treatment of choice is:
(e) Rest and icebag application.
(f) Rest and hot water bag application.
(g) Scrotal support and analgesic.
(h) Surgical exploration.

In colorectal cancers:
(e) At least 50% are in the rectum and sigmoid colon.
(f) Low anterior resection is easier in males than in females.
(g) Most 1cm polyps contain a focus of cancer.
(h) Alteration in bowel habit is common in right sided cancers.
The normal ankle brachial pressure index (ABPI) is:
(e) 0.5
(f) 1
(g) 1.5
(h) 2

Hypersplenism includes all of the following except:


(e) Splenomegaly
(f) Pancytopaenia
(g) Bone marrow hyperplasia
(h) Splenic vein thrombosis

A 43-year-old policeman presents with a painless ulcer over the left medial malleoli with
surrounding pigmentation. The ulcer is likely to be:
(e) Ischaemic ulcer
(f) Venous stasis ulcer
(g) Marjolin’s ulcer
(h) Diabetic ulcer

All of the following factors will delay wound healing except:


(e) Malnutrition
(f) Irradiation
(g) Steroid therapy
(h) Good blood supply

The followings are features of sebaceous cyst except:


(e) Presence of Punctum.
(f) Presence of Sebaceous horn.
(g) Tethering to the skin.
(h) Erosion of the underlying bone

A 25-year-old woman presents to the emergency room complaining of redness


and pain in the right leg. She reports that her right leg has been swollen for the
last 5 years. On examination, her left leg has been normal. She has temperature of 39C and right
leg is not tender but swollen from the inguinal ligament down and she has obvious cellulitis. Her
underlying problem is
(e) Popliteal entrapment syndrome
(f) Acute arterial insufficiency
(g) Primary lymphoedema
(h) Deep vein thrombosis

In full thickness burns of skin:


(e) the whole dermis is destroyed.
(f) it is caused by hot water scalding.
(g) thin-walled painful blisters are usually present.
(h) it heals completely without surgical treatment within 2 weeks.

Burn contractures occur in:


(e) partial thickness burn
(f) whole thickness burn
(g) scalds
(h) frost bite
Regarding medullary carcinoma of thyroid it:
(e) arises from follicular cells.
(f) produces high level of serum calcitonin.
(g) usually occurs in families.
(h) produces high level of tri-iodothyronine.

In hypothyroidism:
(c) pre-tibial myxoedema is present.
(d) it is associated with low serum Thyroid Stimulating Hormone levels.
(c) it occurs immediately after total thyroidectomy.
(d) it occurs immediately after radioiodine therapy

In diffuse toxic goiter :


(e) fine needle aspiration cytology(FNAC) is the investigation of choice.
(f) serum Thyroid Stimulating Hormone is raised.
(g) measurement of serum calcitonin is essential.
(h) oral anti-thyroid drugs can be started immediately upon diagnosis.

Papillary carcinoma of thyroid with lymph node involvement is best treated with :
(e) total thyroidectomy with cervical lymph node dissection.
(f) radio iodine ablation of remnant thyroid tissue.
(g) long-term TSH suppression with oral thyroxine.
(h) all of the above.

A 70 year old lady presents with a progressive enlarging painless hard thyroid nodule, hoarseness of
voice and weight loss. What is the likely diagnosis?
(e) multinodular goitre.
(f) anaplastic carcinoma of thyroid.
(g) follicular adenoma.
(h) de Quervain’s thyroiditis.

1. Severe head injury is considered to be having Glasgow Coma Scale (GSC) score of :
A. more than 12.
B. less than 10.
C. more than 10.
D. less than 8.

2. Blood discharge from the breast nipple is generally seen in :


A. breast abscess.
B. fibroadenoma.
C. duct papilloma.
D. cyclical mastitis (fibrocystic disease of breast).

3. The following factors may adversely affect wound healing, EXCEPT :


A. exposure to ultra-violet radiation.
B. jaundice.
C. advanced neoplasm.
D. uncontrolled diabetes mellitus.

4. Immediate post-operative pyrexia may occur secondary to, EXCEPT :


A. wound infection.
B. urinary tract infection.
C. deep vein thrombosis.
D. subphrenic abscess.

5. Pretibial myxoedema is a feature of :


A. lymphoma of thyroid.
B. myxoedema.
C. Grave’s disease.
D. Reidel’s thyroiditis.

6. Which is LEAST sensitive in detecting obscure gastro-intestinal bleeding ?


A. Barium meal and follow through X-ray.
B. Capsule enteroscopy.
C. Technetium-labeled radionuclide scan.
D. Selective angiography.

7. Complications of blood transfusion include the following, EXCEPT :


A. urticaria.
B. hypovolaemia.
C. Hepatitis C.
D. jaundice.

8. Which of the following is brilliantly transilluminant ?


A. sebaceous cyst.
B. dermoid cyst.
C. lipoma.
D. cystic hygroma.

9. Which of the following is an absorbable suture ?


A. polyglactin (Vicryl).
B. silk.
C. nylon.
D. steel wire.

10. Kehr’s sign is :


A. tenderness at hypochondrium in acute cholecystitis.
B. pain at right iliac fossa on pressing the left iliac fossa in acute appendicitis.
C. pain referred to left shoulder tip in ruptured spleen.
D. ecchymoses around umbilicus in ruptured ectopic pregnancy.

11. The most common organism responsidble for Fournier’s gangrene is :


A. Staphylococcus aureus
B. Escherichia coli.
C. Clostridium welchii.
D. Streptococcus pneumoniae.

12. Which of the following is decreased in acute pancreatitis ?


A. serum amylase.
B. serum lipase.
C. blood glucose.
D. serum calcium.

13. Cullen’s sign in seen in :


A. acute pancreatitis.
B. acute appendicitis.
C. acute cholecystitis
D. acute ischaemic colitis.

14. Perthe’s test is done to test for :


A. incompetence of sapheno-femoral junction.
B. patency of the deep veins.
C. incompetence of popliteal-femoral junction.
D. incompetence of perforators.

15. Portosystemic shunt can be done when :


A. serum albumin is more than 3 gm/ dl.
B. ascites is present.
C. patient is comatose.
D. hepatic encephalopathy is present.

16. The most common site of metastases of colorectal carcinoma is the :


A. right lung.
B. liver.
C. kidney.
D. brain.

17. The following is true of carcinoma of oesophagus :


A. metastases to cervical lymph modes carries poor prognosis.
B. lower third tumour is treated by radiotherapy.
C. initially presents as difficulty in swallowing liquids.
D. adenocarcinoma is more common in the middle third of oesophagus.

18. Risk factors for atherosclerosis includes the following, EXCEPT :


A. smoking.
B. diabetes mellitus.
C. high serum High Density Lipoprotein (HDL)
D. hypertension.

19. Pre-hepatic cause of portal hypertension includes :


A. Budd-Chiari Syndrome.
B. portal vein thrombosis.
C. alcoholic cirrhosis.
D. haemochromatosis.

20. Barrett’s oesophagus is best diagnosed by :


A. ultrasound scan of abdomen.
B. barium meal and follow-through X-rays.
C. endoscopy and biopsy.
D. plain X-ray abdomen.

21. Marjolin ulcer is :


A. a type of acute gastric ulcer associated with major burns injury.
B. an anastomositic ulcer after gastric-intestinal surgery.
C. a variant of syphilitic ulcer.
D. a squamous cell carcinoma in a non-healing ulcer.

22. Sebaceous gland is a type of :


A. apocrine gland.
B. holocrine gland.
C. merocrine gland.
D. endocrine gland.

23. An ulcer with undermined edge is characteristic of :


A. gravitational ulcer.
B. syphilitic ulcer.
C. tuberculous ulcer.
D. trophic ulcer.

24. Basal cell carcinoma :


A. is highly malignant.
B. metastasizes early to regional lymph nodes.
C. is seen in any part of the face.
D. potentially curable with surgery.

25 Wilm’s tumour :
A. is seen in children.
B. is benign in origin.
C. initially presents with haematuria.
D. is treated conservatively.

26. Mammogram :
A. diffentiates between solid lumps and cysts.
B. is not a screening procedure.
C. showing clustered microcalcifications needs biopsy for carcinoma.
D. is painless and has bad effects of irradiation to breasts.

27. Amoeboma is :
A. a chronic amoebic liver abscess.
B. a malignant tumour associated with chronic amoebiasis.
C. a chronic granuloma arising mainly in the colon from amoebiasis.
D. mesenteric lymphadenitis resulting from intestinal amoebiasis.

28. Regarding full thickness burns of skin :


A. It is caused by hot water scalding.
B. The whole dermis is destroyed.
C. It presents as thin-walled painful blisters.
D. It heals completely without surgical treatment within 2 weeks.

29. A major cause of mortality in congenital diaphragmatic hernia is :


A. intestinal obstruction.
B. lung hypoplasia.
C. presence of liver in thoracic cavity.
D. polyhydramnios.

30. The following are congenital abdominal wall defects, EXCEPT :


A. exomphalos minor.
B. exomphalos major.
C. umbilical hernia.
D. ectopia vesicae.

31. Which of the following is true of thyroglossal cyst :


A. It originates from the thyroid gland.
B. It is found in the lateral aspect of the neck.
C. It is a premalignant condition.
D. Its removal involves resection of part of hyoid bone.

32. A 32 year old truck driver presents with a painful fluctuant swelling in his gluteal cleft
for 2 days. What is the most likely diagnosis ?
A. Ischiorectal abscess.
B. Pilonidal abscess.
C. Crohn’s disease of anus.
D. tuberculous ulcer.

33. Femoral hernias :


A. are more common in men.
B. are less likely to strangulate than inguinal hernias.
C. are found medial to the femoral vein.
D. occur as a result of infection.

34. A 25 year old motorcyclist involved in an accident, sustained severe facial jnjuries with
deformity and bleeding . Which of the following measures is correct ?
A. Evaluation of cervical spine should precede the facial injuries management.
B. Direct oral or nasotracheal intubation should performed promptly to prevent airway
obstruction.
C. Standard facial X-ray series are preferable to CT scan to evaluate facial fractures.
D. Definitive management of fractures of facial bones should not be delayed.

1. Which is true about renal-stones?---


a. More common in females.
b. Not found in the urethra.
c. May lead to horse-shoe kidney.
*d. Usually stones are radio-opaque.
2.Best palliative surgery in a case of inoperable
carcinoma of ascending colon---
a. Ileostomy. b. Colostomy in left iliac region.
*c.Ileocolic bye pass anastomosis.
d. Total colectomy.
3. ERCP usually—
a. Requires general anaesthesia.
b. Is contra-indicated in jaundice.
c. Gives same informations as obtained
in PTC(Percutaneous transhepatic cholangiography).
*d. Can diagnose strictures of the pancreatic-duct.

1. A young motorcyclist is thrown against a concrete bridge abutment and sustains severe
trauma to his face with marked deformity and bleeding. Regarding these injuries, which
of the following statements is true ?
A. Evaluation of the cervical spine should precede the management of the facial
injuries.
B. Direct oral or nasotracheal intubation should be performed prompty to prevent
airway obstruction.
C. Standard facial X-rays are preferable to CT scan to assess facial fractures.
D. Definitive management of fractures of facial bones should not be delayed.

2. A 21 year old lady presents with a small painless lump in the midline of her neck which
moves up with swallowing. A clinical diagnosis of thyroglossal cyst was made.
Which of the following statement regarding symptomatic thyroglossal duct cyst is true ?
A. Over 90% manifest itself before age 12.
B. Treatment includes resection of the hyoid bone.
C. They usually present as a painful swelling in the lateral neck.
D. They rarely become infected.

3. A 60 year old smoker presents with an ulcerating mass on his tongue, which on biopsy
was found to be a squamous cell carcinoma. He declines surgical intervention.
Which of the following statements regarding his cancer is true ?
A. Carcinoma of the tongue is best treated by irradiation alone rather than by surgery.
B. Metastases to the pre-auricular lymph nodes is common.
C. Cancer of the tongue most commonly occurs near the frenulum.
D. Local invasion may result in hypoglossal or lingual nerve dysfunction.

4. A 32 year old truck driver presents with a painful fluctuant mass in between gluteal folds
of 2 days duration. Which of the following is the most likely diagnosis ?
A. Fistula in ano.
B. Ischiorectal abscess.
C. Pilonidal abscess.
D. Crohn’s disease of anus.

5. A 47 year old asymptomatic woman was incidentally found to have a 5 mm polyp and no
stones in her gallbladder on ultrasonography scan. Which of the following is the best
management option ?
A. Laparoscopic cholecystectomy.
B. Open cholecystectomy with frozen section biopsy.
C. Observation with repeat ultrasonography scan – to evaluate for increase in
polyp size.
D. En block resection of the gallbladder with wedge resection of liver and portal
lymphadenectomy.

6. Femoral hernias :
A. are more common in men.
B. are found medial to the femoral vein.
C. occur as a result of infection.
D. are less likely to present with incarceration than inguinal hernias.

7. Which of the following is a risk factor for colorectal carcinoma ?


A. Low fat diet.
B. Diverticulosis.
C. Haemorrhoids.
D. Inflammatory bowel disease.

8. The most common site of colorectal metastases is :


A. left lung.
B. liver.
C. brain.
D. left kidney.

9. Following a stab injury to the middle of the upper arm, a 30 year old man cannot flex his
three radial digitals. Which of the following structures has he most likely injured ?
A. Flexor pollicis longus and flexor digitorum longus tendons.
B. Radial nerve.
C. Medial nerve.
D. Ulnar nerve.

10. A 26 year old man presents with urinary burning, frequency and urgency. His external
urinary meatus is inferiorly displaced on his glans. Which of the following statements
about his hypospadias is correct ?
A. It is often associated with cordee.
B. It is often associated with undescended testis (in more than 50% of cases)
C. It is rare fusion defect of the male posterior urethra.
D. The most common location is penoscrotal.

1) Mammogram
A) detects breast carcinoma in young ladies less than 35 years old,
B) differentiates between solid lumps and cysts,
C) shows clustered microcalcifications in carcinoma breast,
E) is a painless procedure and can be repeated often.

2) Radiotherapy in carcinoma of breast


A) is routinely given after simple mastectomy for Paget’s disease,
B) can be given to contralateral breast as prophylaxis,
C) must be given after breast conserving surgery,
D) is safely given in pregnant ladies.
3) Grawitz’s Tumour of kidney,
A) is commonly seen in young children,
B) spreads early to regional lymph nodes,
C) is radiosensitive,
D) transabdominal nephrectomy is the main line of treatment.

4) Basal Cell Carcinoma :


A) is potentially curable with surgery,
B) distant metastases are found early in the disease,
C) spreads early in lymphatics,
D) dark-skinned ladies have a higher risk than fair-skinned men.
1) Wilm’s Tumour :
A) affects the elderly more often than children.
B) smoking is an important cause.
C) is a benign tumour.
D) chemotherapy followed by nephrectomy is treatment.

2) Warthin’s Tumour :
A) is a common tumour in the submandibular gland.
B) is also called adenolymphoma.
C) is often seen in children.
D) invades the facial nerve causing facial palsy.

3) Mumps :
A) is an acute viral infection.
B) affects the elderly more often than children.
C) recurrent parotitis in childhood is an important complication.
D) require antibiotics to treat.
4) Mondor’s disease of breast :
A) is an important pre-requisite to carcinoma.
B) is superficial thrombophlebitis.
C) requires surgery and biopsy.
D) causes nipple discharge.

5) Superficial parotidectomy :
A) usually performed under local anaesthesia.
B) starts with dissection of the anterior border of the parotid gland.
C) location of facial nerve trunk is important.
D) is treatment of choice for high-grade malignant tumours.

6) Urinary bladder diverticulum :


A) laterally located diverticulae are usually congenital.
B) acquired diverticulae are usually caused by bladder outlet obstruction.
C) are detected clinically by a large suprpubic palpable mass.
D) primary resection is necessary in all cases.

7) Grawitz’s tumour of kidney :


A) is an adenocarcinoma.
B) is a transitional cell carcinoma.
C) is more common in women than in men.
D) nephrectomy via loin approach is the choice of treatment.

8) Sentinel node biopsy in carcinoma of breast :


A) is done when no clinically enlarged lymph nodes in the axilla.
B) when there are multiple enlarged palpable axillary lymph nodes.
C) is a risky procedure.
D) can cause lymphoedema of the upper limb.

1 Which of the following occurs in Christmas Disease (deficiency of Factor 9) :


A. Petechial haemorrhages.
B. Haemarthrosis.
C. Spontaneous clotting.
D. Distal embolism.

2 Which transfusion has been notably associated with the transmission of Human
Deficiency Virus (HIV) ?
A. Whole blood.
B. Iron infusion.
C. Packed cells.
D. Anti-haemophilic globulin.

3 Regarding Von Willebrand’s Disease :


A. It is often asymptomatic.
B. Genetically X-linked usually affecting boys.
C. It is often associated with haemarthrosis.
D. It is often associated with thrombocytopaenia.

4 Stored blood transfusions :


A. are given for thrombocytopaenia.
B. can transmit Hepatitis A virus.
C. can transmit Hepatitis B virus.
D. are given when haemoglobin level is12 mg/ 100 mls or less.

5 Treatment with oral iron sulphate for anaemia causes :


A. haemolytic jaundice.
B. indigestion.
C. pale coloured stools.
D. spherocytic red blood cells.

6 Rapid transfusions of stored blood may produce :


A. hypothermia.
B. hypercalcaemia.
C. dark stools.
D. tea-coloured urine.

7 Hyperaldosteronism is diagnosed by :
A. a reduced ratio between renin and aldosterone.
B. a high blood pressure reduced by alpha-blockers (phenoxybenzamine).
C. base excess of minus 10.
D. sudden increases of blood pressure.

8 The signs of a tumour secreting vasoactive intestinal polypeptide (VIP) include :


A. hypoglycaemia.
B. watery diarrhoea.
C. peptic ulcers.
D. hypercalcaemia.

9 Persistent duodenal ulceration even after vagotomy and partial gastrectomy is a feature
of :
A. Zollinger-Ellison Syndrome (primary hypergastrinaemia)
B. a Vipoma.
C. Conn’s Syndrome.
D. acromegaly.

10 Episodic hypertension and glycosuria is associated with :


A. Type 1 diabetes mellitus.
B. Multiple Endocrine Neoplasia (MEN) Type 1.
C. phaeochromocytoma.
D. insulinoma.

11 Primary hyperparathyroidism is most commonly associated with :


A. a solitary parathyroid adenoma.
B. parathyroid hyperplasia.
C. renal failure.
D. Multiple Endocrine Neoplasia (MEN) Type 1.

12 The features of Zollinger-Ellison Syndrome include :


A. high acids and low gastrins.
B. ulcers in the fundus of the stomach.
C. no control of acid secretion by proton-pump inhibitors.
D. basal acid output of over 60% of maximal acid output.

13 Cushing’s Syndrome is typically caused by :


A. adenocortical hyperplasia.
B. adenoma of posterior lobe of pituitary gland.
C. adenoma of anterior lobe of pituitary gland.
D. hyperaldosteronism.

14 Pancreatic tumours which may present with gain in weight include :


A. glucagonoma.
B. insulinoma.
C. pancreatic ductal cancer.
D. vipoma.

15 Which of the following is associated with pituitary tumours :


A. bilateral hemianopia.
B. bilateral sensory deafness.
C. episodes of watery diarrhoea.
D. hypokalaemia.

16 What percentage of people with primary hepatocellular carcinoma worldwide survive


five years ?
A. 90 %.
B. 50 %.
C. 30 %.
D. 10 %.

17 In hepatocellular carcinoma (HCC), which factor is most commonly present ?


A. A preceding Hepatitis A infection.
B. A preceding Hepatitis C infection.
C. Cirrhosis of liver.
D. Gallstones.

18 What percentage of bile salts in bile have been recycled ?


A. 5 %.
B. 10 %.
C. 20 %.
D. 80 %

19 Why is Entamoeba histolytica and not Entamoeba dispar, a pathogen ?


A. It moves at 10 um./ second.
B. It resists dessication.
C. It possesses an adhesion molecule gag/ Gal Nac.
D. It does not use the Emden-Meyerof pathway.

20 How is amoebic abscess best treated ?


A. Repeated aspiration and injection of a scolicidal agent.
B. Metronidazole 800 mg TDS for 10 days orally.
C. Metronidazole 800 mg TDS for 10 days orally and paromomycin.
D. Dihydroemetine therapy.

21 What complications may arise from a hydatid cyst of the liver ?


A. Bacteraemia.
B. Rupture into pericardium.
C. Gastric fistula.
D. Aplastic anaemia.

22 Inoperable hepatocellular carcinoma giving rise to symptoms may be treated by :


A. Radiotherapy to the liver.
B. 5-fluorouracil biweekly intravenous injections.
C. Division of the portal vein.
D. Trans-arterial chemotherapy embolisation (TACE)

23 The commonest benign tumour of the skin is :


A. keratoacanthoma.
B. senile keratosis.
C. Merkel tumour.
D. Hutchinson’s frekle.

24 Keratoacanthoma is :
A. a self-limiting condition.
B. usually takes six months to develop.
C. locally invasive.
D. caused by a Streptococcal viridians infection.

25 Skin grafts on the face usually :


A. are taken from the thigh skin.
B. are taken from the arm skin.
C. are full skin grafts ideally,
D. are generally split skin grafts.

26 Squamous cell carcinoma :


A. commonly metastasizes by blood stream.
B. is common on the lower limb.
C. is associated histologically with keratin pearls.
D. is an autosomal recessive condition.

27 Basal cell cancer (rodent ulcer) :


A. has a homogenous histology.
B. metastasizes frequently.
C. arises from the basal cells of the epidermis.
D. is common on the lower limbs.

28 Malignant melanoma.
A. is common in Africans.
B. becomes more frequent after exposure to sunlight.
C. Clark’s level 5 is associated with 30% five-year survival.
D. Clark’s level 1 is associated with 60% five-year survival.
29 Kaposi’s sarcoma :
A. is only seen in patients with Acquired Immune Deficiency Syndrome (AIDS)
B. is a tumour of smooth muscle.
C. is caused by Herpes virus 8.
D. is best treated by radiotherapy.

1 Marjolin ulcer is

A a type of acute gastric ulcer associated with major burn injury.

B an ulcer at an anastomosis after prior gastric surgery.

C a gastric vascular malformation.

D associated with squamous cell carcinoma in a chronic wound.

2 Dermoid cyst is

A commonly found in the mid line of the body.

B lined by columnar epithelium.

C adhered to the overlying skin.

D transillumination postive.

3 Sebaceous gland is

A apocrine type.

B holocrine type.

C merocrine type.

D endocrine type.

B
4 A 38 year-old lady presents to the emergency department with pain on defaecation lasting
about one hour. She also has noted some bright red blood on the tissue paper. The most
likely diagnosis is:

A Bleeding internal haemorhoids

B Thrombosed external haemorrhoids

C Proctitis

D Anal fissure

5 Single fistula-in-ano is commonly seen following

A Crohn’s disease.

B Colloid carcinoma of rectum.

C Perianal abscess.

D Tuberculosis

6 Vaginal hydrocoele is a type of:

A retention cyst.

B exudation cyst.

C cystic tumour.

D degenerative cyst.

7 The following conditions are congenital abdominal wall defects except

A Exomphalos minor

B Exomphalos major

C Umbilical hernia

D Ectopia Vesicae

8 An ulcer with an undermined edge is seen in

A gravitational ulcer.

B syphilitic ulcer.
C tuberculous ulcer.

D trophic ulcer.

9 Major cause regarding the mortality in congenital diaphragmatic hernia is due to:

A Size of the hiatus of the hernia.

B lung hypoplasia.

C presence of liver in the thoracic cavity.

D polyhydramnios.

10 62-year-old man with a history of smoking presents with hematuria. He has owned and
operated a chain of dry cleaners for over 30 years. The most likely diagnosis is:

A Prostate cancer

B Renal cell carcinoma

C Bladder cancer

D Carcinoma of the ureter

10 The followings are predisposing factors for carcinoma of penis.


11 A 26 year old lady notices a change in appearance of a mole on her neck. Which of the
following
A does not indicate malignant change ?
leukoplakia.

A
B Pallor of disease.
Bowen’s the lesion.

B
C Irregularity
early of surface.
circiumcision.

C
D Bleeding
penile on touch.
warts.

D Satellite lesions besides the original mole.

12 Marginal ulcer is :

A an ulcer in the stomach found in associated with brain tumour.

B an ulcer in the stomach associated with burns.

C an ulcer in the stomach associated with head injury.

D an ulcer at an anastomotic site after previous gastric surgery.

13 In mechanical intestinal obstruction, abdominal distension is unlikely to be caused by :


A swallowed air.

B carbon monoxide gas produced in the bowel.

C increased intestinal secretion proximal to obstruction.

D decreased intestinal absorption proximal to obstruction.

14 The most common location of thyroglossal cyst is :

A sublingual.

B suprahyoid.

C over the thyroid cartilage.

D subhyoid.

15 The strongest layer of small intestines is :

A serosa..

B mucosa.

C submucosa.

D muscularis mucosa.

16 A 68 yer old lady who had been treated for a static leg ulcer for the past 5 years developed a
noduleat one side of the ulcer over 2 weeks which started to bleed intermittently and then
ulcerate itself.The most likely diagnosis is :

A spreading venous ulcer.

B rodent ulcer.

C diabetic ulcer.

D Marjolin’s ulcer.

1) 30 year old lady comes with pain upper abdomen, fever, chills and rigors and with
icterus - what should be the first line of investigation ? :
A ) liver function test
B ) total haemogram
C ) ultrasound of hepato-biliary region
D ) CT scan of abdomen
2) 50 year old alcoholic, suddenly after meal, develops acute pain in abdomen with
fever, tachycardia, hypotension – likely diagnosis should be ? :
A ) acute gastro-enteritis
B ) acute pancreatitis
C ) acute cholecystitis
D ) acute mesenteric vascular ischaemia

3) 25 year old male reports with complaints of feeling heaviness of this right testis.
On examination, the testis is found to be hard in consistency with mild hydrocoele of
right scrotum. No inguinal lymphadenopathy and no history of trauma.
The preferred management should be :
A ) fine needle aspiration cytology (FNAC) of right testis
C ) open biopsy of right testis
D ) scrotal incision and removal of right testis
E ) inguinal incision and removal of right testis

1 Sentinel node biopsy in carcinoma of breast usually involves biopsy of lymph node in :
A. ipsilateral axilla..
B. contralateral axilla.
C. the supraclavicular region.
D. the internal mammary area.

2 When a clinical diagnosis of acute intestinal obstruction is made, the next appropriate
imaging procedure is ?
A. Barium contrast studies of upper gastrointestinal tract.
B. Plain supine and erect views of abdomen.
C. Computerized tomography of abdomen and pelvis.
D. Magnetic resonance imaging of abdomen.

3 Fournier’s gangrene :
A. is cused by thrombosis of spermatic arteries.
B. is a dry gangrene.
C. is often associated with diabetes mellitus.
D. will heal well with antibiotic therapy alone.

4 Bilateral gynaecomastia :
A. is a premalignant condition.
B. will always show lactation.
C. can be seen in chronic liver disease.
D. presents with bloody nipple discharge.

5 Intestinal amoebiasis :
A. is caused by Entamoeba coli.
B. causes carcinoma of colon.
C. can lead to liver abscess.
D. causes intestinal obstruction.

6 Parathormone produces its effects through :


A. direct action on bone leading to bone resorption
B. decreased calcium absorption from intestine
C. increased renal tubular resorption of phosphate
D. decreased renal tubular resorption of calcium

7 Strangulated intestinal obstruction is seen in :


A. tuberculosis of terminal ileum.
B. Crohn’s disease.
C. acute mesenteric arterial thrombosis.
D. acute intussusception.

8 In acute arterial occlusion of a limb :


A. it is characterized by a warm limb.
B. treatment is immediate use of vasodilator drug.
C. irreversible muscle necrosis occurs after 6 hours.
D. arterial bypass graft is done as the appropriate surgical treatment

9 An ulcer by definition is :
A. an abrasion of the skin.
B. a breach in the continuity of an epithelial surface.
C. a defect caused by infection.
D. a bleeding malignant lesion.

10 Ludwig’s angina :
A. is an infection of closed fascial space in the neck.
B. is a form of oesophageal motility disorder.
C. is an abscess above the palatine tonsil.
D. is associated with coronary artery spasm.

32 Best palliative surgery in a case of inoperable carcinoma of ascending colon is :


A. ileostomy.
B. colostomy in left iliac region.
C. ileocolic bypass anastomosis.
D. total colectomy.

33 ERCP usually :
A. requires general anaesthesia.
B. can diagnose strictures of the pancreatic duct.
C. is contra-indicated in jaundice.
D. gives same information as obtained in Percutaneous transhepatic cholangiography
(PTC).

34 Which of the following is true about urinary stones ?


A. More common in females.
B. Not found in the urethra.
C. Usually stones are radio-opaque.
D. May lead to horse-shoe kidney.

35 A 45 year old lady presented with bloody discharge from left nipple for 3 weeks with a
small non-tender firm lump in her left breast. What is a possible diagnosis ?
A. galactocoele.
B. duct papilloma.
C. acute breast abscess.
D. fibroadenoma breast.

36 A 53 year old lady who was diagnosed with early stage carcinoma of caecum.
During operation the following structures that were removed included
A. distal 20 cm of ileum.
B. lymph nodes along left colic artery.
C. whole descending colon
D. right ovary and Fallopian tube.

37 A 55 year old lady sustained extensive full-thickness burns to her right leg 5 years ago
was left with an ulcer on her right leg just above the ankle which seemed to be larger
recently. Which of the following procedures should be undertaken ?
A. Continue with Eusol dressings and wearing of compression stockings.
B. Take edge biopsy for the possibility of Marjolin’s ulcer.
C. Give a combination of wide spectrum antibiotics for 2 weeks.
D. Perform split skin grafting on ulcer.

38 A baby was born with complete cleft lip and cleft palate.
Appropriate management of this case would be :
A. CT scan done to assess other skull abnormalities.
B. Primary closure of cleft palate is done first before closure of cleft lip.
C. Primary closure of cleft palate is done by 2 months old.
D. Primary closure of cleft lip is done at 3 months old.

39 Regarding hepatocellular carcinoma :


A. risk factors include previous viral hepatitis A infection.
B. serum alpha-fetoprotein is a useful tumour marker.
C. majority of cases are curable with hepatic resection.
D. tumour metastases to regional lymph nodes is common.

40 A 65 year old man presented with acute prolapsed irreducible haemorrhoids.


Appropriate management of this case includes :
A. Barron’s banding is done for the external haemorrhoids.
B. Colonoscopy is done first to rule out colorectal carcinoma
C. Forceful reduction of the haemorrhoids under sedation.
D. Oral medication given as out-patient treatment.

1) Barrett’s oesophagus is lined by :


A) squamous epithelium.
B) intestinal metaplasic epithelium.
C) columnar epithelium.
D) cuboidal epithelium.

2) Barrett’s oesophagus is :
A) a premalignant lesion of the oesophagus.
B) is a frank malignant lesion of the oesophagus.
C) is due to a viral infection.
D) is due to duodenal ulceration.

Cullen’s sign in acute pancreatitis is :


A : ecchymoses at the sides of abdominal wall.
B : ecchymoses around the umbilicus.
C : rebound tenderness at epigastrium.
D : pain referred to the left shoulder.

In perforated peptic ulcer, pneumoperitoneum can be demonstrated clinically as :


A : shifting dullness on percussion.
B : succussion splash in upper abdomen.
C : loss of liver dullness on percussion.
D : loud bowel sounds on auscultation.

1 Grawitz tumour of kidney


A is an adenocarcinoma.

B presents with bilateral loin masses.

C is more common in women than in men.

D nephrectomy via loin approach is the choice of treatment.

2 Sentinel node biopsy in carcinoma of breast :


A is a risky procedure.

B is done on multiple lymph nodes in the axilla.

C can cause lymphaedema of upper limb.

D is done when there are no clinically enlarged axillary lymph nodes.

3 The following are true of basal cell carcinoma, EXCEPT :


A usually arises from skin of the face.

B is potentially curable by wide surgical resection.

C spreads early by lymphatics.

D light skinned men have a higher risk than dark skinned women.

4 Rapid transfusions of stored blood may produce :


A hypothermia.

B hypercalcaemia.

C dark coloured stools.

D thrombocytosis.

5 Episodic hypertension and glycosuria is associated with :


A insulinoma.

B phaechromocytoma.

C medullary carcinoma of thyroid.

D craniopharyngioma.
6 In hepatocellular carcinoma (HCC), which factor is most commonly present ?
A A preceding Hepatitis A infection.

B amoebic abscess of liver.

C cirrhosis of liver.

D gallstones.

7 In colorectal carcinoma, which is the commonest site ?


A Caecum.

B Transverse colon.

C Sigmoid colon.

D Rectum.

8 Xerostomia is caused by :
A post-irradiation to head and neck.

B pleomorphic adenoma of parotid gland.

C stone in submandibular duct.

D ranula.

9 Of the matching pairs of tumour marker to malignancy, which of the following is TRUE :
A serum prostatic specific antigen – carcinoma bladder.

B serum human gonadotrophin – carcinoma of cervix.

C serum calcitonin – medullary carcinoma of thyroid.

D serum alphafetoprotein – carcinoma colon.

10 Definitive treatment of carcinoma of lower third of oesophagus is


A total oesophagectomy.

B total oesophago-gastrectomy.

C radiotheraphy.

D chemotherapy.

1) A gallbladder tumour will present as a palpable abdominal mass in the :


A) right hypochondrium.
B) left hypochondrium.
C) right iliac fossa.
D) left iliac fossa.

2) Gallstones is formed from the constituents of :


A) gastric juice.
B) pancreatic juice.
C) succus entericus.
D) bile.

3) In obstructive jaundice is characterized by :


A) increase in unconjugated bilirubin
B) decrease in unconjugated bilirubin.
C) increase in conjugated bilirubin.
D) decrease in conjugated bilirubin.

4) The following are complications of gallstones, except :


A) acute pancreatitis.
B) empyema of gallbladder.
C) chronic cholecystitis.
D) hepatocellular carcinoma.

5) An early sign of carcinoma head of pancreas is :


A) large mass in left hypochondrium.
B) obstructive jaundice.
C) diabetes mellitus.
D) malabsorption.

1 Components of the balanced anaesthesia include


A. loss of consciousness

B. analgesia.

C. Muscle relaxation

D. apnoea

2 During preoperative airway assessment,one must look for

A. limitation of mouth opening.

B. a receding mandible.

C. size of the lip.

D. position,number and health of teeth.

3 The purposes of giving premedication are to achieve

A. anxiolysis.

B. basal narcosis.

C. anti-emesis.

D. anti-tussive.

4 Common indications for endotracheal intubation are

A. where muscle relaxants are used to facilitate surgery like laparotomy.

B. patients with full stomach.

C. when the insertion of LMA is failed.

D. operations on the head and neck.

5 Complications of endotracheal intubation are

A. hypoxia due to unrecognized oesophageal intubation.

B. hyperglycemia.

C. hypertension and arrhythmias .

D. laryngeal spasm.

6 The use of muscle relaxant is to

A. aid endotracheal intubation.

B. reduce the maintenance dose of anaesthetic agents.


C. help surgery.

D. reduce postoperative pain.

7 Postoperative recovery room discharge criteria include the followings

A. the ability to maintain and protect airway.

B. sufficient postoperative pain relief.

C. haemodynamic stability.

D. return of the bowel sound.

8 Contraindications to spinal anaesthesia include

A. severe coagulopathy.

B. infection near the puncture site.

C. chronic obstructive pulmonary disease.

D. elevated intracranial pressure.

9 The drug that is used to treat benzodiazepine overdose is

A. Naloxone.

B. Flumazenil.

C. Naltrexone.

D. Morphine sulphate.

10 Visual analogue scale is used to measure

A. sleep

B. sedation

C. depth of anaesthesia

D. pain intensity

1 Regarding morphine

A. It causes histamine release.(T)

B. It has no active metabolites.(F)

C. It undergoes extensive first pass hepatic metabolism. (T)


D. It has an elimination half-life of 3-4 hours. (T)

2 Regarding propofol

A. It causes smooth and rapid induction of anaesthesia. (T)

B. Recovery is also rapid. (T)

C. It can cause postoperative nausea and vomiting.(F)

D. It causes pain at the site of injection. (T)

3 Regarding the systemic toxicity of local anaesthetic agents

A. Maximum dose of bupivacaine is 2mg/kg BW with or without adrenaline. (T)

B. Accidental intravascular injection causes systemic local toxicity. (T)

C. Circumoral numbness is the early sign of systemic local toxicity. (T)

D. Vasodilators are used to treat systemic local toxicity.(F)

4 Regarding the management of pain

A. Visual Analogue Scale (VAS) is widely used to assess the severity of pain. (T)

B. WHO analgesic ladder is used in the management of cancer pain. (T)

C. Antipsychotic is one of the drugs used to treat pain.(F)

D. Opoids are used to treat moderate to severe pain. (T)

You might also like